You are on page 1of 58

Mathematics (JEE-Advanced) Application of Derivative

APPLICATION OF DERIVATIVE
TANGENT & NORMAL
I The value of the derivative at P (x1 , y1) gives the
slope of the tangent to the curve at P. Symbolically
dy 
f (x1) =  = Slope of tangent at
dx  x
1 y1

P (x1 y1) = m (say).

II Equation of tangent at (x1, y1) is ;


dy 
y  y1 =  (x  x1).
dx  x
1 y1

III Equation of normal at (x1, y1) is ;


1
y  y1 =  (x  x1).
dy 
dx  x
1 y1

Ex. The equation of the tangent to the curve x2 (x – y) + a2 (x + y) = 0 at origin is-


(A) x + y + 1 = 0 (B) x + y + 2 = 0 (C) x + y = 0 (D) 2x – y = 0
Sol. 3 2 2
The given equation of the curve is x – x y + a x + a y = 0 2

Differentiating it w.r.t. x
dy dy
3x2 – 2xy – x2. + a2 + a2. =0
dx dx
dy dy
– x2. + a2 . dx = – 3x2 + 2xy – a2
dx
Now at origin i.e. x = 0, y = 0
 a2 (1 + dy/dx) = 0
 dy/dx = –1
 the equation of tangent is
y – 0 = – 1 (x – 0) y = – x  x + y = 0
Ex. The point of contact on the curve x + y = a where-
(i) tangent line is parallel to x - axis
(ii) tangent line is parallel to y - axis
(iii) tangent line makes equal angles with both the axis.
(A) (a,0), (0,a), (a/4, a/4) (B) (0,a), (a, 0), (a/4, a/2)
(C) (0,a), (a/4, a/4), (a,a) (D) (0,0), (a,0), (0,a)
Sol. Differentiating equation of the curve w.r.t. x
y
dy/dx = –
x
(i) If tangent line is parallel to x - axis, then
dy/dx = 0  y = 0 and x = a
Thus the point is (a,0)
(ii) If tangent is parallel to y – axis , then
dy/dx =   x = 0 and y = a

Campus-I : F-106, Road No.2, Indraprastha Industrial Area, Kota Ph : 2420151 / 52, 9166876626
Campus-II : A-10, Jawahar Nagar Main Road, Kota Ph : 2406123 / 24
1
Mathematics (JEE-Advanced) Application of Derivative
Thus the point is (0,a)
(iii) If tangent line makes equal angles with both axis , then dy/dx =  1
 y = x = a/4
Thus the point is (a/4, a/4)
Ex. The length of perpendicular from (0,0) to the normal drawn at point (1,6) to the curve
y = 2x2 + 3x + 1 is-
(A) 1 (B) 1/2 (C) 43/ 50 (D) 1/6

FG dy IJ
x1  y1
H dx K
Sol.  P' =
F dy I
1 G J
2

H dx K
Since dy/dx = 7
1  6.7
 P' = = 43
1  49 50
Ex. The abscissa of a point where the normal drawn to the curve xy = (x+ c)2 makes an equal
intercept with coordinate axes is-
c
(A)  2 c (B)  (C)  3 c (D)  3 c
2 2
Sol. Differentiating the given curve w.r.t. x
dy
x + y = 2(x + c)
dx
dy 2 ( x  c)  y
 =
dx x
2 ( x1  c)  y1
 = 1 and x1y1 = (x1 + c)2
x1

2 ( x1  c)  1
x c b
2
g 2 2
x1 2 x1  2 c x1  x1  2c x1  c 2
= = 1 = 2 = 1
x1 x1

2
x1  c 2
= 2 = 1  x12 – c2 =  x12  x12 – c2 = x12
x1
c
and 2x12 – c2 = 0  2x12 = c2  x1 = 
2
Ex. The equation of the tangent to the curve y = be–x/a at the point where it meets y– axis is-
x x x y x y x y
(A) + =1 (B) + =1 (C) + = 2 (D) + =2
b b a b b a a b
Sol. The point of intersection of the curve with
y- axis is (0,b).Differentiating y = be–x/a we have
dy b –x/a FG dy IJ b
dx
=–
a
e  H dx K ( 0, b)
=–
a
 the equation of the required tangent is
b x y
y–b=– (x– 0)  + =1
a a b

Campus-I : F-106, Road No.2, Indraprastha Industrial Area, Kota Ph : 2420151 / 52, 9166876626
Campus-II : A-10, Jawahar Nagar Main Road, Kota Ph : 2406123 / 24
2
Mathematics (JEE-Advanced) Application of Derivative
1
Ex. Find the equation of all straight lines which are tangent to curve y = and which are parallel to
x 1
the line x + y = 0.
Sol.: Suppose the tangent is at (x1, y1) and it has slope – 1.
dy
 = – 1.
dx ( x1 , y1 )

1
 – ( x  1)2 = – 1.
1

 x1 = 0 or 2

 y1 = – 1 or 1
Hence tangent at (0, – 1) and (2, 1) are the required lines (see figure) with equations
– 1(x – 0) = (y + 1) and – 1 (x – 2) = (y – 1)
 x+y+1=0 and y+x=3
Ex. Find equation of normal to the curve y = |x2 – | x | | at x = – 2.
Sol. In the neighborhood of x = – 2, y = x2 + x.
Hence the point of contact is (– 2, 2)
dy dy
= 2x + 1  = – 3.
dx dx x  2
So the slope of normal at (– 2, 2) is 1/3.
Hence equation of normal is
1
(x + 2) = y – 2  3y = x + 8.
3
Ex. Prove that sum of intercepts of the tangent at any point to the curve x + y = a on the
coordinate axis is constant.
Sol. Let P(x1, y1) be a variable point on the curve x + y = a , as shown in figure.
y1
 equation of tangent at point P is – (x – x1) = (y – y1)
x1

x y
 – x1
+ x1 = y1
– y1

x y
 x1 + y1 = x1 + y1

x y
 x1 + y1 = a ( x1  y1  a )

Hence point A is  ax , 0 and coordinates of point B is (0,


1
ay1 ) . Sum of intercepts

= a ( x1 + y1 ) = a . a = a (which is constant)
Ex. Tangent at P(2, 8) on the curve y = x3 meets the curve again at Q. Find coordinates of Q.
Sol. Equation of tangent at (2, 8) is
y = 12x – 16
Solving this with y = x3
x3 – 12x + 16 = 0
This cubic will give all points of intersection of line and curve
y = x3 i.e., point P and Q. (see figure)
But, since line is tangent at P so x = 2 will be a repeated root of

Campus-I : F-106, Road No.2, Indraprastha Industrial Area, Kota Ph : 2420151 / 52, 9166876626
Campus-II : A-10, Jawahar Nagar Main Road, Kota Ph : 2406123 / 24
3
Mathematics (JEE-Advanced) Application of Derivative
3
equation x – 12x + 16 = 0 and another root will be x = h. Using theory of equations :
sum of roots  2+2+h=0  h=–4
Hence coordinates of Q are (– 4, – 64)
NOTE :
1. The point P (x1 , y1) will satisfy the equation of the curve & the equation of tangent & normal line.
2. If the tangent at any point P on the curve is parallel to the axis of x then dy/dx = 0 at the point P.
3. If the tangent at any point on the curve is parallel to the axis of y, then dy/dx =  or dx/dy = 0.
4. If the tangent at any point on the curve is equally inclined to both the axes then dy/dx = ± 1.
5. If the tangent at any point makes equal intercept on the coordinate axes then dy/dx = – 1.
6. Tangent to a curve at the point P (x1, y1) can be drawn even through dy/dx at P does not exist.
e.g. x = 0 is a tangent to y = x1/3 at (0, 0).
7. If a curve passing through the origin be given by a rational integral algebraic equation, the equation of the
tangent (or tangents) at the origin is obtained by equating to zero the terms of the lowest degree in the
equation.
e.g. If the equation of a curve be x2 – y2 + x3 + 3 x2 y  y3 = 0, the tangents at the origin are given by
x2 – y2 = 0 i.e. x + y = 0 and x  y = 0.

IV Angle of intersection between two curves is defined as the angle between the 2 tangents drawn to the
2 curves at their point of intersection. If the angle between two curves is 90° every where then they are
called ORTHOGONAL curves.
x2 y2
Ex The condition that the curves 2 – = 1 and xy = c2 mutually intersect orthogonally is -
a b2

a2
(A) a2 + b2 = 0 (B) a2 = b2 (C) a2 – b2 = 0 (D) =1
b2
Sol. The given curves are
x2 y2
– =1 ... (1) and xy = c2 ... (2)
a2 b2

F dy I
from (1), GH dx JK
b2 x FG dy IJ
1
= 2
a y
and
2
H K
from (2) , dx = – y/x
(1) and (2) intersect orthogonally if
FG dy IJ FG dy IJ F b xI F yI
2

H dx K H dx K =–1  GH a y JK GH  x JK
2 = – 1  a2 = b2 which is the required condition
1 2

Ex. Find the angle between curves y2 = 4x and y = e–x/2


Sol. Let the curves intersect at point (x1, y1) (see figure).
dy 2
for y2 = 4x, dx = y
( x1, y1 ) 1

dy 1 x / 2 y1
and for y = e–x/2 dx =– e 1 =–
( x1, y1 ) 2 2
 m1 m2 = – 1 Hence  = 90°

Note that we have not actually found the intersection point but geometrically we can see that the curves
intersect.

Campus-I : F-106, Road No.2, Indraprastha Industrial Area, Kota Ph : 2420151 / 52, 9166876626
Campus-II : A-10, Jawahar Nagar Main Road, Kota Ph : 2406123 / 24
4
Mathematics (JEE-Advanced) Application of Derivative
y1 1  f (x1 )
2
y1
V (a) Length of the tangent (PT) = (b) Length of Subtangent (MT) =
f ( x1 ) f ( x1 )
(c) Length of Normal (PN) = y1 1  f ( x1 )2 (d) Length of Subnormal (MN) = y1 f ' (x1)

Ex. Find the length of tangent, sub-tangent, normal and sub-normal at the point  of the curve x =
a (  + sin  ), and y = a(1–cos  ).
FG dy IJ bdy / dg a sin  dy 2 sin  / 2 cos  / 2
Sol.  H dx K =
bdx / dg b
= a 1 cos  g 
dx
=
2 cos 2  / 2
dy 
 = tan  length of tangent
dx 2

a sin 2  / 2 sec  / 2
= b
a 1  cos  g 1  tan 2  / 2 = sin  / 2
= a. sin  / 2
tan  / 2
cos  / 2
length of normal
= a (1–cos) 1  tan 2  / 2 = a.2 sin2  / 2 . sec  / 2
length of sub tangent

=
b
a 1  cos  g =
2 a sin 2  / 2
= a sin 
tan  / 2 sin  / 2
cos  / 2

length of sub normal = a (1–cos  ) tan  /2 = 2a sin2  /2 tan  /2

Ex. Find possible values of p such that the equation px2 = nx has exactly one solution.
Sol. Two curves must intersect at only one point.

(i) (ii)

I. If p  0 then there exists only one solution (see graph - (i))


II. If p > 0
then the two curves must only touch each other
i.e. tangent at y = px2 and y = nx must have same slope at point (x1, y1)
1 1
 2px1 = x  x12 = 2p .........(i)
1

1  
also y1 = px12  y1 = p  2p 
 
1 1
 y1 = ........(ii) and y1 = nx1  = nx1
2 2
 x1 = e1/2 .........(iii)
1 1 1
 x12 = 2p  e = 2p  p=
2e

 1
Hence possible values of p are (– , 0]   2e 
 

Campus-I : F-106, Road No.2, Indraprastha Industrial Area, Kota Ph : 2420151 / 52, 9166876626
Campus-II : A-10, Jawahar Nagar Main Road, Kota Ph : 2406123 / 24
5
Mathematics (JEE-Advanced) Application of Derivative
3 2
Ex. Find the length of tangent for the curve y = x + 3x + 4x – 1 at point x = 0.
dy
Sol. Here, m = dx
x0

dy
= 3x2 + 6x + 4  m=4
dx
and, k = y(0)  k=–1
1 1 17
 = |k| 1  2   = | ( 1) | 1  =
m 16 4
Ex. For the curve y = a n (x2 – a2) show that sum of lengths of tangent & subtangent at any point
is proportional to coordinates of point of tangency.
Sol. Let point of tangency be (x1, y1)
dy 2ax1
m = dx =
x  x1 x 21  a2

1 y1
Length of tangent + subtangent = |y1| 1  2 +
m m

( x 21  a 2 )2 2
y 1( x 1  a 2 ) 4
x1  a 4  2a 2 x1
2 2
y 1( x 1  a 2 )
= |y1| 1 2 + = |y1| +
4a 2 x 1 2ax 1 2 | ax1 | 2ax 1

2 2
y 1( x 1  a 2 ) y 1( x 1  a 2 ) | y | ( 2x ) x y
2
= 2ax 1 + 2ax 1 = 21 | ax 1| = 1a 1
1

VI DIFFERENTIALS :
The differential of a function is equal to its derivative multiplied by the differential of the independent
variable. Thus if, y = tan x then dy = sec2 x dx. In general dy = f  (x) d x.
Note that : d (c) = 0 where 'c' is a constant.
d (u + v  w) = du + dv  dw d (u v) = u d v + v d u
Note :
1. For the independent variable 'x' , increment  x and differential d x are equal but this is not the case with
the dependent variable 'y' i.e.  y  d y.
dy
2. The relation d y = f  (x) d x can be written as = f  (x) ; thus the quotient of the differentials of 'y' and
dx
'x' is equal to the derivative of 'y' w.r.t. 'x'.
ROLLE'S THEOREM :
Let f(x) be a function of x subject to the following conditions :
(i) f(x) is a continuous function of x in the closed interval of a  x  b.
(ii) f  (x) exists for every point in the open interval a < x < b.
(iii) f (a) = f (b).
Then there exists at least one point x = c such that a < c < b where f  (c) = 0.
Note that if f is not continuous in closed [a, b] then it may lead to the adjacent
graph where all the 3 conditions of Rolles will be valid but the assertion will
not be true in (a, b).

Ex. Verify Rolle’s theorem for f(x) = (x – a)n (x – b)m , where m, n are positive real numbers, for
x  [a, b].
Sol. Being a polynomial function f(x) is continuous as well as differentiable. Also f(a) = f(b)
 f(x) = 0 for some x  (a , b)
n(x – a)n–1 (x – b)m + m(x – a)n (x – b)m–1 = 0
Campus-I : F-106, Road No.2, Indraprastha Industrial Area, Kota Ph : 2420151 / 52, 9166876626
Campus-II : A-10, Jawahar Nagar Main Road, Kota Ph : 2406123 / 24
6
Mathematics (JEE-Advanced) Application of Derivative
n–1 m–1
 (x – a) (x – b) [(m + n) x – (nb + ma)] = 0
nb  ma
 x= , which lies in the interval (a, b), as m, n  R+.
mn
Ex. If 2a + 3b + 6c = 0 then prove that the equation ax2 + bx + c = 0 has at least one real root
between 0 and 1.
ax 3 bx 2
Sol. Let f(x) = + + cx
3 2
a b
f(0) = 0 and f(1) = + + c = 2a + 3b + 6c = 0
3 2
If f(0) = f(1) then f(x) = 0 for some value of x  (0, 1)
 ax2 + bx + c = 0 for at least one x  (0, 1)
LMVT THEOREM :
Let f(x) be a function of x subject to the following conditions :
(i) f(x) is a continuous function of x in the closed interval of a  x  b.
(ii) f  (x) exists for every point in the open interval a < x < b.
(iii) f(a)  f(b).
f ( b)  f (a)
Then there exists at least one point x = c such that a < c < b where f  (c) =
ba
Geometrically, the slope of the secant line joining the curve at x = a & x = b is equal to the slope of the
tangent line drawn to the curve at x = c. Note the following :
 Rolles theorem is a special case of LMVT since
f ( b)  f (a)
f (a) = f (b)  f  (c) = = 0.
ba
Ex. Verify LMVT for f(x) = – x2 + 4x – 5 and x  [–1, 1]
Sol. f(1) = –2 ; f(–1) = –10
f (1)  f (1)
 f(c) = 1  ( 1)  –2c + 4 = 4  c=0
ba ba
Ex. Using Lagrange’s mean value theorem, prove that if b > a > 0, then < tan–1 b – tan–1 a <
1 b2 1  a2
Sol. Let f(x) = tan–1 x ; x  [a, b] applying LMVT
tan 1 b  tan 1 a 
f(c) = for a < c < b and f(x) = ,
ba 1 x 2
Now f(x) is a monotonically decreasing function
Hence if a < c < b  f(b) < f(c) < f(a)
1 tan 1 b  tan 1 a 1
 2 < < Hence proved
1 b ba 1  a2

Ex. Let f : R  R be a twice differentiable function such that f(2) = 8, f(4) > 64, f(7) = 343 then show that
there exists a c  (2, 7) such that f(c) < 6c.
Sol. Consider g(x) = f(x) – x3
By LMVT
g (4)  g (2) g (7 )  g ( 4 )
= g(c1) , 2 < c1 < 4 and = g(c2), 4 < c2 < 7
42 74
g(c1) > 0 , g(c2) < 0
By LMVT
g (c 2 )  g (c 1 )
c 2  c1
= g(c), c1 < c < c2
 g(c) < 0
 f(c) – 6c < 0 for same c  (c1 , c2) c (2, 7)
Campus-I : F-106, Road No.2, Indraprastha Industrial Area, Kota Ph : 2420151 / 52, 9166876626
Campus-II : A-10, Jawahar Nagar Main Road, Kota Ph : 2406123 / 24
7
Mathematics (JEE-Advanced) Application of Derivative
Note : Now [f (b) – f (a)] is the change in the function f as x changes from a to b so that [f (b) – f (a)] / (b – a)
is the average rate of change of the function over the interval [a, b]. Also f '(c) is the actual rate of
change of the function for x = c. Thus, the theorem states that the average rate of change of a function
over an interval is also the actual rate of change of the function at some point of the interval. In particular,
for instance, the average velocity of a particle over an interval of time is equal to the velocity at some
instant belonging to the interval.
This interpretation of the theorem justifies the name "Mean Value" for the theorem.

MONOTONICITY
(Significance of the sign of the first order derivative)
DEFINITIONS :
1. A function f (x) is called an Increasing Function at a point x = a if in a sufficiently small neighbourhood
f (a  h)  f (a) and 
around x = a we have  increasing;
f (a  h)  f (a) 
f (a  h)  f (a) and 
Similarly decreasing if  decreasing.
f (a  h)  f (a) 
2. A differentiable function is called increasing in an interval (a, b) if it is increasing at every point within the
interval (but not necessarily at the end points).A function decreasing in an interval (a, b) is similarly
defined.
3. A function which in a given interval is increasing or decreasing is called “Monotonic” in that interval.
4. Tests for increasing and decreasing of a function at a point :
If the derivative f (x) is positive at a point x = a, then the function f (x) at this point is increasing. If it is
negative, then the function is decreasing. Even if f'(a) is not defined, f can still be increasing or decreasing.

Note : If f (a) = 0, then for x = a the function may be still increasing or it may be decreasing as shown. It has
to be identified by a seperate rule. e.g. f (x) = x3 is increasing at every point.
Note that, dy/dx = 3 x².

Ex. Function f(x) = cos x – 2x is decreasing when -


(A)  > 1/2 (B)  < 1/2 (C)  < 2 (D)  > 2
Sol. f(x) is monotonic decreasing when
f' (x) < 0  x
 –sinx – 2 < 0  2 > – sin x
 2 > 1   > 1/2
(  maximum value of – sin x = 1)
Ex.6 The function f(x) = [x(x –3)]2 is increasing when –
(A) 0 < x <  (B) –  < x < 0 (C) 0 < x < 3/2 (D) 1 < x < 3
Sol. We have f(x) = [x(x –3)] 2

f'(x) = 2x(x – 3) [ (x – 3) + x ]
= 2x (x –3) (2x – 3)
Campus-I : F-106, Road No.2, Indraprastha Industrial Area, Kota Ph : 2420151 / 52, 9166876626
Campus-II : A-10, Jawahar Nagar Main Road, Kota Ph : 2406123 / 24
8
Mathematics (JEE-Advanced) Application of Derivative
If f(x) is an increasing function, then
f'(x) > 0  x(x –3) (2x – 3) > 0  0 < x < 3/2 or x > 3
Ex. Function f(x) = sin x – cos x is monotonic increasing when –
(A) x  (0, /2) (B) x  ( –/4, /4) (C) x  (/4, 3/4) (D) No where
FG 1 cos x  1 IJ
Sol. f'(x) = cos x + sin x = 2
H 2 2
sin x
K = 2 sin(/4 + x)
Now f(x) is monotonic increasing when f'(x) > 0
 2 sin(/4 + x) > 0  0 < /4 + x < 
(  sin is positive when 0 <  < )  x  (–/4, 3/4)

Ex. f(x) = 2x2 – log | x | (x  0) is monotonic increasing in the interal –


(A) (1/2, ) (B) ( –, –1/2)  (1/2, )
(C) ( –, –1/2)  (0, 1/2) (D) (–1/2, 0)  (1/2, )
Sol. f'(x) = 4x – 1/x
f(x) is monotonic increasing when f'(x) > 0

4 x2  1
 4x – 1/x > 0  >0
x
4x2 – 1 > 0 when x > 0
4x2 – 1 < 0 when x < 0
But x > 0, 4x2 – 1 > 0  x2 > 1/4  |x| > 1/2
 x  (1/2, )
But x < 0, 4x –1< 0  x2 < 1/4  |x| < 1/2
2

 x  ( –1/2, 0)
 x  ( –1/2, 0)  (1/2, )
Ex. Let f(x) = x – sinx. Find the intervals of monotonicity.
Sol. f(x) = 1 – cosx
Now, f(x) > 0 every where, except at x = 0, ± 2, ± 4 etc. But all these points are discrete
(countable) and do not form an interval. Hence we can conclude that f(x) is strictly increasing in
R. In fact we can also see it graphically.

Ex Find the intervals of monotonicity of the following functions.


(i) f(x) = x2 (x – 2)2 (ii) f(x) = x n x
(iii) f(x) = sinx + cosx ; x  [0, 2]
2 2
Sol. (i) f(x) = x (x – 2)
f(x) = 4x (x – 1) (x – 2)
observing the sign change of f(x)

Hence increasing in [0, 1] and in [2, )

Campus-I : F-106, Road No.2, Indraprastha Industrial Area, Kota Ph : 2420151 / 52, 9166876626
Campus-II : A-10, Jawahar Nagar Main Road, Kota Ph : 2406123 / 24
9
Mathematics (JEE-Advanced) Application of Derivative
and decreasing for x  (– , 0] and [1, 2]
(ii) f(x) = x n x
f(x) = 1 + n x
1
f(x)  0  n x  – 1  x
e
1 1
 increasing for x   ,   and decreasing for x   0 ,  .
e   e
(iii) f(x) = sinx + cosx
f(x) = cosx – sinx
for increasing f(x)  0  cosx  sinx
5
 f is increasing in 0 ,   and  , 2
 4   4 

 5 
f is decreasing in  , 
4 4 
Note : If a function f(x) is increasing in (a, b) and f(x) is continuous in [a, b], then f(x) is increasing on [a, b]
Ex. f(x) = [x] is a step up function. Is it a strictly increasing function for x  R.
Sol No, f(x) = [x] is increasing (monotonically increasing) (non-decreasing), but not strictly increasing
function as illustrated by its graph.

Ex. If f(x) = sin4x + cos4x + bx + c, then find possible values of b and c such that f(x) is monotonic
for all x  R
Sol. f(x) = sin4x + cos4x + bx + c
f(x) = 4 sin3x cosx – 4cos3x sinx + b = – sin4x + b.
Case - (i) : for M.I. f(x)  0 for all x  R
 b  sin4x for all x  R  b1
Case - (ii) : for M.D. f(x)  0 for all x  R
 b sin4x for all x  R  b–1
Hence for f(x) to be monotonic b  (– , – 1]  [1, ) and c  R.
Ex. Find possible values of 'a' such that f(x) = e2x – (a + 1) ex + 2x is monotonically increasing for
xR
Sol. f(x) = e2x – (a + 1) ex + 2x
f(x) = 2e2x – (a + 1) ex + 2
Now, 2e2x – (a + 1) ex + 2  0 for all x  R
x 1 
 2 e   – (a + 1)  0 for all x  R
 ex 

x  1 
(a + 1) < 2  e   for all x  R
 ex 
 1 
 a+14  ex  x
has min imum value 2 
 e 
 a3
Campus-I : F-106, Road No.2, Indraprastha Industrial Area, Kota Ph : 2420151 / 52, 9166876626
Campus-II : A-10, Jawahar Nagar Main Road, Kota Ph : 2406123 / 24
10
Mathematics (JEE-Advanced) Application of Derivative
Aliter (Using graph)
2e2x – (a + 1) ex + 2  0 for all x  R
putting ex = t ; t  (0, )
2t 2 – (a + 1) t + 2  0 for all t  (0, )

Case - (i) : D0


 (a + 1)2 – 4  0
 (a + 5) (a – 3) 0
 a  [– 5, 3]
or
Case - (ii) : both roots are non positive

b
D0 & – <0 & f(0)  0
2a
a1
 a  (– , – 5]  [3, ) & <0 & 20
4
 a  (– , – 5]  [3, ) & a<–1 & aR
 a  (– , – 5]
Taking union of (i) and (ii), we get a  (– , 3].
5. Tests for Increasing & Decreasing of a function in an interval :
SUFFICIENCY TEST : If the derivative function f (x) in an interval (a , b) is every where positive, then the
function f (x) in this interval is Increasing ;
If f (x) is every where negative, then f (x) is Decreasing.
Ex. In which interval the function f(x) = x2 – x + 1 is not a monotonic function –
(A) (0, 1/2) (B) (1/2, ) (C) (0, 1) (D) None of these
Sol. Here f'(x) = 2x –1
Obviously f(x) is monotonic decreasing function in the interval (0, 1/2) (since x < 1/2  2x – 1
< 0) and is monotonically increasing in the interval (1/2, 1)
Thus the function is neither a decreasing function nor an increasing function in (0, 1)
Ex. In the interval (1, 2), function f(x) = 2 |x – 1| + 3| x – 2| is –
(A) Monotonic increasing (B) Monotonic decreasing
(C) Not monotonic (D) Constant
Sol. x  (1, 2)  f(x) = 2(x –1) – 3(x – 2) = –x + 4
 f'(x) = –1 < 0  x
 f(x) is monotonic decreasing in (1, 2)
Ex. If f(x) = x2 + kx + 1 is increasing function in the interval [ 1, 2], then least value of k is–
(A) 2 (B) 4 (C) –2 (D) –4
Sol. f' (x) = 2x + k
 f(x) is increasing function in the interval [1, 2]
 f' (x) > 0  2x + k > 0  k > –2x,  x  [1, 2]
or k > –2x, when 1  x  2 or k > –2 and –4
 k  max (–2, –4)  k  –2
Therefore least value of k is –2.
Ex. If y = ax3 + 3x2 + (2a + 1)x + 1000 is strictly increasing function for all values of x, then –
(A) –3/2 < a < 1 (B) a > 1 (C) a < –3/2 (D) a > 1 or a < – 3/2

Campus-I : F-106, Road No.2, Indraprastha Industrial Area, Kota Ph : 2420151 / 52, 9166876626
Campus-II : A-10, Jawahar Nagar Main Road, Kota Ph : 2406123 / 24
11
Mathematics (JEE-Advanced) Application of Derivative
Sol. y is increasing function
 3ax2 + 6x + (2a + 1) > 0  x  36 – 4.3a(2a + 1) < 0 and a > 0
 3 – 2a2 – a < 0 and a > 0  (2a + 3) (a – 1) > 0 and a > 0
 a > 1 or a < –3/2 and a > 0  a>1
Ex. If a function f(x) = cos | x | – 2ax + b is an increasing function on whole number line, then the value
of a is –
b 1 3
(A) b (B) (C) a   (D) a > 
2 2 2
d
Sol.  cos| x | = – sin x, for x  R
dx
 f' (x) = – sin x – 2a
Now f(x) is an increasing function, therefore
1 1
f' (x) > 0  – sin x – 2a > 0    a <  sin x  a  
2 2

x3 x3  tan 1 x 
Ex. For x  (0, 1) prove that x – –1
< tan x < x – lim
hence or otherwise find x0  x 
3 6  

x3
Sol. Let f(x) = x – – tan–1x
3
1
f(x) = 1 – x2 –
1 x2

x4
f(x) = –
1 x2
f(x) < 0 for x  (0, 1)  f(x) is M.D.
x3
 f(x) < f(0)  x– – tan–1x < 0
3
x3
 x– < tan–1x ...........(i)
3
x3
Similarly g(x) = x – – tan–1x
6
x2 1
g(x) = 1 – –
2 1 x2

x 2 (1  x 2 )
g(x) =
2(1  x 2 )
g(x) > 0 for x  (0, 1)  g(x) is M.I.
 g(x) > g(0)
x3
x– – tan–1x > 0
6

x3
x– > tan–1x ........(ii)
6
from (i) and (ii), we get
x3 x3
x– < tan–1x < x – Hence Proved
3 6

Campus-I : F-106, Road No.2, Indraprastha Industrial Area, Kota Ph : 2420151 / 52, 9166876626
Campus-II : A-10, Jawahar Nagar Main Road, Kota Ph : 2406123 / 24
12
Mathematics (JEE-Advanced) Application of Derivative
x2 tan 1 x x2
Also, 1 – < <1– , for x > 0
3 x 6
tan 1 x
Hence by sandwich theorem we can prove that xlim
0
= 1 but it must also be noted that
x
tan 1 x tan 1 x
as x  0, value of  1 from left hand side i.e. <1
x x

 tan 1 x 
 lim   =0
x 0
 x 
NOTE : In proving inequalities, we must always check when does the equality takes place because the
point of equality is very important in this method. Normally point of equality occur at end point of
the interval or will be easily predicted by hit and trial.
  x3
Ex. For x   0, 2  , prove that sin x > x –
  6

x3
Sol. Let f(x) = sin x – x +
6

x2
f(x) = cos x – 1 +
2
we cannot decide at this point whether f(x) is positive or negative, hence let us check for
monotonic nature of f(x)
f(x) = x – sinx
 
Since f(x) > 0  f(x) is M.I. for x   0, 2 
 
 f(x) > f(0)
 f(x) > 0  f(x) is M..
 f(x) > f(0)
x3
 sin x – x + >0
6

x3
 sin x > x – Hence proved
6
 sin x tan x 
Ex. Examine which is greater : sin x tan x or x2. Hence evaluate xlim
0   , where
 x2 
 
x   0, 2 
 
Sol. Let f(x) = sinx tanx – x2
f(x) = cos x . tan x + sin x . sec2x – 2x
 f(x) = sin x + sin x sec2x – 2x
 f(x) = cos x + cos x sec2x + 2sec2x sin x tan x – 2
 f(x) = (cos x + sec x – 2) + 2 sec2x sin x tan x
 
 
Now cos x + sec x – 2 = cos x  sec x 2 and 2 sec2x tan x . sin x > 0 because x   0, 2 
 
 f(x) > 0  f(x) is M.I.
Hence f(x) > f(0)
 f(x) > 0  f(x) is M.I.

Campus-I : F-106, Road No.2, Indraprastha Industrial Area, Kota Ph : 2420151 / 52, 9166876626
Campus-II : A-10, Jawahar Nagar Main Road, Kota Ph : 2406123 / 24
13
Mathematics (JEE-Advanced) Application of Derivative
2
 f(x) > 0  sin x tan x – x > 0
Hence sin x tan x > x2
sin x tan x 

sin x tan x
>1  lim   =1
x 2 x 0  x2 
Ex. Compare which of the two is greater (100)1/100 or (101)1/101.
Sol Assume f(x) = x1/x and let us examine monotonic nature of f(x)
 1  nx 
f(x) = x1/x .  
 x2 
f(x) > 0  x  (0,e)
and f(x) < 0  x  (e,)
Hence f(x) is M.D. for x e
and since 100 < 101
 f(100) > f(101)
 (100)1/100 > (101)1/101
General Note :
(1) If a continuous function is invertible it has to be either increasing or decreasing.
(2) If a function is continuous the intervals in which it rises and falls may be separated by points at which its
derivative fails to exist.
(3) If f is increasing in [a, b] and is continuous then f (b) is the greatest and f (c) is the least value of f in
[a, b]. Similarly if f is decreasing in [a, b] then f (a) is the greatest value and f (b) is the least value.
6. APPLICATION OF ROLLES THEOREM FOR ISOLATING THE REAL ROOTS OF AN
EQUATION f (x)=0
Suppose a & b are two real numbers such that ;
(i) f(x) & its first derivative f  (x) are continuous for a  x  b.
(ii) f(a) & f(b) have opposite signs.
(iii) f  (x) is different from zero for all values of x between a & b.
Then there is one & only one real root of the equation f(x) = 0 between a & b.

MAXIMA - MINIMA
FUNCTIONS OF A SINGLE VARIABLE
HOW MAXIMA & MINIMA ARE CLASSIFIED
1. A function f(x) is said to have a maximum
at x = a if f(a) is greater than every other
value assumed by f(x) in the immediate
neighbourhood of x = a. Symbolically
f (a)  f (a  h)
 x = a gives maxima
f (a)  f (a  h)
for a sufficiently small positive h.
Similarly, a function f(x) is said to have a
minimum value at x = b if f(b) is least than
every other value assumed by f(x) in the
immediate neighbourhood at x = b.
Symbolically if
f ( b)  f ( b  h)
 x = b gives minima for a sufficiently small positive h.
f (b)  f ( b  h)

Campus-I : F-106, Road No.2, Indraprastha Industrial Area, Kota Ph : 2420151 / 52, 9166876626
Campus-II : A-10, Jawahar Nagar Main Road, Kota Ph : 2406123 / 24
14
Mathematics (JEE-Advanced) Application of Derivative
| x | 0  | x |  2
Ex. Let f(x) =  1 x  0 . Examine the behaviour of f(x) at x = 0.

Sol. f(x) has local maxima at x = 0 (see figure).

 (b 3  b 2  b  1)
 – x3  0  x 1
Ex. Let f(x) =  (b 2  3b  2)
2x  3 1 x  3

Find all possible values of b such that f(x) has the smallest value at x = 1.
Sol. Such problems can easily be solved by graphical approach (as in figure).

Hence the limiting value of f(x) from left of x = 1 should be either greater or equal to the value of
function at x = 1.
lim f(x)  f(1)
x 1

(b 3  b 2  b  1) (b 2  1)(b  1)
 –1+ –1  0
(b 2  3b  2) (b  1) (b  2)
 b  (– 2, –1)  [1, + )
Note that :
(i) the maximum & minimum values of a function are also known as local/relative maxima or
local/relative minima as these are the greatest & least values of the function relative to some neighbourhood
of the point in question.
(ii) the term 'extremum' or (extremal) or 'turning value' is used both for maximum or a minimum value.
(iii) a maximum (minimum) value of a function may not be the greatest (least) value in a finite interval.
(iv) a function can have several maximum & minimum values & a minimum value may even be greater than
a maximum value.
(v) maximum & minimum values of a continuous function occur alternately & between two consecutive
maximum values there is a minimum value & vice versa.
2. A NECESSARY CONDITION FOR MAXIMUM & MINIMUM :
If f(x) is a maximum or minimum at x = c & if f  (c) exists then f  (c) = 0.
Note :
(i) The set of values of x for which f  (x) = 0 are often called as stationary points or critical points. The rate
of change of function is zero at a stationary point.
(ii) In case f  (c) does not exist f(c) may be a maximum or a minimum & in this case left hand and right hand
derivatives are of opposite signs.
(iii) The greatest (global maxima) and the least (global minima) values of a function f in an interval [a, b] are
f(a) or f(b) or are given by the values of x for which f  (x) = 0.
dy
(iv) Critical points are those points in the domain of function where either = 0 or it fails to exist.
dx

Campus-I : F-106, Road No.2, Indraprastha Industrial Area, Kota Ph : 2420151 / 52, 9166876626
Campus-II : A-10, Jawahar Nagar Main Road, Kota Ph : 2406123 / 24
15
Mathematics (JEE-Advanced) Application of Derivative
3 2
Ex. If f(x) = x + ax + bx + c has extreme values at x = – 1 and x = 3. Find a, b, c.
Sol. Extreme values basically mean maximum or minimum values, since f(x) is differentiable function so
f(– 1) = 0 = f(3)
f(x) = 3x2 + 2ax + b
f(3) = 27 + 6a + b = 0
f(– 1) = 3 – 2a + b = 0
 a = – 3, b = – 9, c  R
Ex. Find the critical points of the function f(x) = 4x3 – 6x2 – 24x + 9 if
(i) x  [0, 3] (ii) x  [–3, 3] (iii) x  [– 1, 2].
Sol. f(x) = 12(x2 – x – 2)
= 12(x – 2) (x + 1)
f(x) = 0  x = – 1 or 2
(i) if x  [0, 3] , x = 2 is critical point.
(ii) if x  [– 3, 3], then we have two critical points x = – 1, 2.
(iii) If x  [– 1, 2], then no critical point as both x = 1 and x = 2 become boundary points.
3. SUFFICIENT CONDITION FOR EXTREME VALUES :
f  (c- h) > 0 
 x = c is a point of local maxima, where f  (c) = 0.  h is a sufficiently
f  (c+ h) < 0  small positive
f  (c- h) < 0   quantity
Similarly  x = c is a point of local minima, where f (c) = 0.
f  (c+ h) > 0
Note : If f  (x) does not change sign i.e. has the same sign in a certain complete neighbourhood of c,
then f(x) is either strictly increasing or decreasing throughout this neighbourhood implying that f(c) is not
an extreme value of f.
Ex. Find the points of maxima, minima of f(x) = x3 – 12x. Also draw the graph of this functions.
Sol. f(x) = x3 – 12x
f(x) = 3(x2 – 4) = 3(x – 2) (x + 2)
f(x) = 0  x=±2

For tracing the graph let us find maximum and minimum values of f(x).

x f (x)
2  16
 2  16

Ex. Show that f(x) = (x3 – 6x2 + 12x – 8) does not have any point of local maxima or minima. Hence
draw graph
Sol. f(x) = x3 – 6x2 + 12x – 8
f(x) = 3(x2 – 4x + 4)
f(x) = 3(x – 2)2
f(x) = 0  x=2
but clearly f(x) does not change sign about x = 2.
f(2+) > 0 and f(2– ) > 0. So f(x) has no point of
maxima or minima. In fact f(x) is a monotonically
increasing function for x  R.

Campus-I : F-106, Road No.2, Indraprastha Industrial Area, Kota Ph : 2420151 / 52, 9166876626
Campus-II : A-10, Jawahar Nagar Main Road, Kota Ph : 2406123 / 24
16
Mathematics (JEE-Advanced) Application of Derivative
3 2 2
Ex. Let f(x) = x + 3(a – 7)x + 3(a – 9) x – 1. If f(x) has positive point of maxima, then find possible
values of 'a'.
Sol. f(x) = 3 [x2 + 2(a – 7)x + (a2 – 9)]
Let ,  be roots of f(x) = 0 and let  be the smaller root. Examining sign change of f(x).

Maxima occurs at smaller root  which has to be positive. This basically implies that both
roots of f(x) = 0 must be positive and distinct.
29
(i) D>0  a<
7

b
(ii) – >0  a<7
2a
(iii) f(0) > 0  a  (– , – 3) (3, )
 29 
from (i), (ii) and (iii)  a  (– , – 3)   3, 
 7 

Ex. Find critical points of f(x) = max (sinx, cosx) , x  (0, 2).
Sol.

  5
From the figure it is clear that f(x) has three critical points x = , , .
4 2 4
Ex. Find the possible points of Maxima/Minima for f(x) = |x2 – 2x| (x  R)
x 2  2 x x  2
 2
Sol. f(x) = 2x  x 0  x  2
x 2  2 x x  0

2( x  1) x  2

f(x) = 2(1  x ) 0  x  2
2( x  1) x  0

f(x) = 0 at x = 1 and f(x) does not exist at x = 0, 2. Thus these are critical points.

4. USE OF SECOND ORDER DERIVATIVE IN ASCERTAINING THE MAXIMA OR


MINIMA:
(a) f(c) is a minimum value of the function f, if f  (c) = 0 & f  (c) > 0.
(b) f(c) is a maximum value of the function f, f  (c) = 0 & f  (c) < 0.
Note : if f  (c) = 0 then the test fails. Revert back to the first order derivative check for ascertaning the
maxima or minima.

Campus-I : F-106, Road No.2, Indraprastha Industrial Area, Kota Ph : 2420151 / 52, 9166876626
Campus-II : A-10, Jawahar Nagar Main Road, Kota Ph : 2406123 / 24
17
Mathematics (JEE-Advanced) Application of Derivative
Ex. Find the points of local maxima or minima for f(x) = sin2x – x, x  (0, ).
Sol. f(x) = sin2x – x
f(x) = 2cos2x – 1
1  5
f(x) = 0  cos 2x =  x= ,
2 6 6
f(x) = – 4 sin 2x
 
f  6  < 0  Maxima at x =
  6

 5  5
f  6  > 0  Minima at x =
  6
Ex. Find points of local maxima or minima of f(x) = x5 – 5x4 + 5x3 – 1
Sol. f(x) = x5 – 5x4 + 5x3 – 1
f(x) = 5x2 (x – 1) (x – 3)
f(x) = 0  x = 0, 1, 3
2
f(x) = 10x (2x – 6x + 3)
Now, f(1) < 0  Maxima at x = 1
f(3) > 0  Minima at x = 3
and, f(0) = 0  nd derivative test fails
so, f(x) = 30 (2x2 – 4x + 1)
f(0) = 30
 Neither maxima nor minima at x = 0.
Note : It was very convenient to check maxima/minima at first step by examining the sign
change of f(x) no sign change of f(x) at x = 0
f(x) = 5x2 (x – 1) (x – 3)

Ex.. The minimum value of the function xx (x > 0) is at -


(A) x =1 (B) x = e (C) x = e–1 (D) None of these
d
Sol. Let y = xx  log y = x log x  (log y) = 1 + log x
dx
2
d 1
and 2 (log y) = = x–1
dx x
Now for minimum value of y or log y
d
(log y) = 0  1 + log x = 0  x = e–1 Again for x = e–1
dx
d2
(log y) = e>0  y is minimum at x = e–1
dx2
Ex. If x = p and x = q are respectively the maximum and minimum points of the function
x5 – 5x4 +5x3 – 10, then-
(A) p=0, q= 1 (B) p = 1, q = 0 (C) p = 1, q = 3 (D) p = 3, q = 1
Sol. Let f(x) = x – 5x + 5x3 – 10, then
5 4

f' (x) = 5x4 – 20 x3 + 15x2


= 5x2 (x–1) (x–3)
and f"(x) = 20x3 – 60 x2 + 30 x
For maxima and minima
f'(x) = 0  5x2 ( x– 1) (x–3) = 0
 x = 0, 1,3 Also f" (1) = – 10 < 0  x = 1 is a point of maxima  p = 1
and f"(3) = 90 > 0  x= 3 is a point of minima  q = 3.
Campus-I : F-106, Road No.2, Indraprastha Industrial Area, Kota Ph : 2420151 / 52, 9166876626
Campus-II : A-10, Jawahar Nagar Main Road, Kota Ph : 2406123 / 24
18
Mathematics (JEE-Advanced) Application of Derivative
40
Ex. The minimum value of the function is-
3 x  8x  18x2  60
4 3

(A) 2/3 (B) 3/2 (C) 40/53 (D) None of these


1 dy 1
Sol. Let y = (3x4 + 8x3 – 18 x2 + 60)  = (12x3 + 24x2 – 36x)
40 dx 40
d2 y 1
and 2 = (36x2 + 48x –36)
dx 40
dy
Now = 0 x3 + 2x2 – 3x = 0
dx
or x(x –1) (x + 3) = 0 or x = 0, 1, – 3
d2 y
At x = 0, = – 36 < 0  y is maximum at x = 0
dx2
1
 the given function i.e. y is minimum at x = 0.
40 2
 minimum value of the function = =
60 3
Ex. f(x) = 1+ 2 sin x + 3 cos2 x (0  x  2/3) is-

(A) minimum at x = /2 (B) maximum at x = sin–1 (1/ 3 )


(C) minimum at x = /3 (D) minimum at x = sin–1 (1/3)
Sol. f'(x) = 2 cos x – 6 cos x sin x
f" (x) = – 2 sin x + 6 sin2 x – 6 cos2x
= – 2 sin x + 12 sin2 x – 6
Now f '(x) = 0  cos x = 0 and sin x = 1/3 or x =  /2 & x = sin–1 (1/3)
so f " (  /2) = – 2 + 12 – 6 > 0
FG
1 1 2 4IJ
H
f" sin 3 =
3 K
+ – 6 < 0
3
 f(x) is minimum at x =  /2.

2
 2 x1) sin2 x
Ex. The minimum value of e(2x is-
(A) e (B) 1/e (C) 1 (D) 0
and u = (2x2 – 2x – 1) sin2 x
2
 2 x 1) sin2 x
Sol. Let y = e(2 x
du
Now
dx

= (2x2 –2x –1) 2sin x cos x+(4x– 2) sin2x


= sin x [2(2x2 – 2x ) cos x + (4x –2) sin x]
du
= 0  sin x = 0  x = n 
dx
d 2u d
= sin x [2(2x2–2x–1) cos x + (4x– 2) sin x] + cos x [2 cos x(2x2–2x–1) + (4x–2) sin x]
dx 2 dx

d 2u
At x = n  , = 0 + 2 cos2 n  (2n2  2–2n  –1)>0
dx 2
Hence at x = n  , the value of u and so its corresponding the value of y is minimum and
minimum value = eº = 1.

Campus-I : F-106, Road No.2, Indraprastha Industrial Area, Kota Ph : 2420151 / 52, 9166876626
Campus-II : A-10, Jawahar Nagar Main Road, Kota Ph : 2406123 / 24
19
Mathematics (JEE-Advanced) Application of Derivative
5. SUMMARYWORKING RULE :
FIRST :
When possible , draw a figure to illustrate the problem & label those parts that are important in the
problem. Constants & variables should be clearly distinguished.
SECOND :
Write an equation for the quantity that is to be maximised or minimised. If this quantity is denoted by
‘y’, it must be expressed in terms of a single independent variable x. his may require some algebraic
manipulations.
THIRD :
If y = f (x) is a quantity to be maximum or minimum, find those values of x for which
dy/dx = f (x) = 0.
FOURTH :
Test each values of x for which f(x) = 0 to determine whether it provides a maximum or minimum or
neither. The usual tests are :
(a) If d²y/dx² is positive when dy/dx = 0  y is minimum.
If d²y/dx² is negative when dy/dx = 0  y is maximum.
If d²y/dx² = 0 when dy/dx = 0, the test fails.
positive for x  x 0 
dy 
(b) If is zero for x  x 0  a maximum occurs at x = x0.
dx
negative for x  x 0 
But if dy/dx changes sign from negative to zero to positive as x advances through
xo there is a minimum. If dy/dx does not change sign, neither a maximum nor a minimum. Such points
are called INFLECTION POINTS.
FIFTH :
If the function y = f (x) is defined for only a limited range of values a  x  b then examine x = a &
x = b for possible extreme values.
SIXTH :
If the derivative fails to exist at some point, examine this point as possible maximum or minimum.
 x2  x ; 1 x  0

  ; x0
Ex. Let f(x) = 
  1 3
log1/ 2  x   ; 0  x 
  2 2

Discuss global maxima, minima for  = 0 and = 1. For what values of  does f(x) has global
maxima
Sol Graph of y = f(x) for  = 0

–1/2 3/2
–1

–1

No global maxima, minima


Graph of y = f(x) for  = 1
1

Global maxima is 1, which occurs at x = 0


–1/2 3/2
Global minima does not exists –1
Lim f(x) = 0, Lim f(x) = 1, f(0) = 
x 0  x 0 –1

For global maxima to exists


f(0)  1    1.
Campus-I : F-106, Road No.2, Indraprastha Industrial Area, Kota Ph : 2420151 / 52, 9166876626
Campus-II : A-10, Jawahar Nagar Main Road, Kota Ph : 2406123 / 24
20
Mathematics (JEE-Advanced) Application of Derivative
40
Ex. Find extrema of f(x) = 3x4 + 8x3 – 18x2 + 60. Draw graph of g(x) = and comment on its local and
f(x)
global extrema.
Sol. f(x) = 0
 12x (x2 + 2x – 3) = 0
 12x (x – 1) (x + 3) = 0
 x = –3, 0, 1
+ +
f(x) = 12(x + 3) x(x – 1) –
–3 0

1
local minima occurs at x = –3, 1
local maxima occurs at x = 0
f(–3) = – 75, f(1) = 53 are local minima
f(0) = 60 is local maxima
Lim f(x) =  , Lim f(x) = 
x  x  

Hence global maxima does not exists : Global minima is – 75


40 60
g(x) = (f ( x ))2 f(x)
53
 g(x) has same critical points as that of f(x). –3
A rough sketch of y = f(x) is   1
Let zeros of f(x) be , 
g(), g() are undefined, – 75

Lim g(x) =  , xLim g(x) = –  , Lim g(x) = –  , xLim


 
g(x) = 
x     x 

x =  , x =  are asymptotes of y = g(x).


Lim g(x) = 0, Lim g(x) = 0
x  x  

 y = 0 is also an asymptote.
1
 x = –3, x = 1 are local minima of 40/53
y = f(x)  –3
2/3
x = – 3, x = 1 are local maxima of y = g(x)   1
–(8/15)
similarly, x = 0 is local minima of y = g(x)
Global extrema of g(x) does not exists.
A rough sketch of y = g(x) is
Important Note :
– Given a fixed point A(x1, y1) and a moving point P(x, f (x)) on the curve y = f(x). Then AP will be
maximum or minimum if it is normal to the curve at P.
– If the sum of two positive numbers x and y is constant than their product is maximum if they are
equal, i.e. x + y = c , x > 0 , y > 0 , then
1
xy = [ (x + y)2 – (x – y)2 ]
4
– If the product of two positive numbers is constant then their sum is least if they are equal.
i.e. (x + y)2 = (x – y)2 + 4xy
6. USEFUL FORMULAE OF MENSURATION TO REMEMBER :
 Volume of a cuboid = lbh.  Surface area of a cuboid = 2 (lb + bh + hl).
 Volume of a prism = area of the base x height.
 Lateral surface of a prism = perimeter of the base x height.
 Total surface of a prism = lateral surface + 2 area of the base
(Note that lateral surfaces of a prism are all rectangles).
1
 Volume of a pyramid = area of the base x height.
3

Campus-I : F-106, Road No.2, Indraprastha Industrial Area, Kota Ph : 2420151 / 52, 9166876626
Campus-II : A-10, Jawahar Nagar Main Road, Kota Ph : 2406123 / 24
21
Mathematics (JEE-Advanced) Application of Derivative
1
 Curved surface of a pyramid = (perimeter of the base) x slant height.
2
(Note that slant surfaces of a pyramid are triangles).
1
 Volume of a cone =  r2h.
3
 Curved surface of a cylinder = 2  rh.

 Total surface of a cylinder = 2  rh + 2  r2.


4
 Volume of a sphere =  r3.
3
 Surface area of a sphere = 4  r2.
1 2
 Area of a circular sector = r  , when  is in radians.
2
Ex. If the equation x3 + px + q = 0 has three real roots, then show that 4p3 + 27q2 < 0.
Sol. f(x) = x3 + px + q, f(x) = 3x2 + p
 f(x) must have one maximum > 0 and one minimum < 0. f(x) = 0
p
 x=± ,p0
3

p p
f is maximum at x = – and minimum at x =
3 3

  p    p 
f  – f 0
 3   3 

  
 q  2p  p   q  2p  p   0
 3 3   3 3 

4p3
q2 + < 0, 4p3 + 27q2 < 0.
27
Ex. Find two positive numbers x and y such that x + y = 60 and xy3 is maximum.
Sol x + y = 60
 x = 60 – y  xy3 = (60 – y)y3
3
Let f(y) = (60 – y) y ; y  (0, 60)
for maximizing f(y) let us find critical points
f(y) = 3y2 (60 – y) – y3 = 0
f(y) = y2 (180 – 4y) = 0
 y = 45
f(45+) < 0 and f(45– ) > 0. Hence local maxima at y = 45.
So x = 15 and y = 45.
Ex. Rectangles are inscribed inside a semicircle of radius r. Find the rectangle with maximum area.
Sol. Let sides of rectangle be x and y (as shown in figure).
 A = xy.
Here x and y are not independent variables and are related by Pythogorus theorem with r.
x2 x2
+ y2 = r2  y = r2 
4 4
x2
 A(x) = x r 2 
4
x4
 A(x) = x 2r 2 
4
Campus-I : F-106, Road No.2, Indraprastha Industrial Area, Kota Ph : 2420151 / 52, 9166876626
Campus-II : A-10, Jawahar Nagar Main Road, Kota Ph : 2406123 / 24
22
Mathematics (JEE-Advanced) Application of Derivative
4
x
Let f(x) = r2x2 – ; x  (0, r)
4
A(x) is maximum when f(x) is maximum
Hence f(x) = x(2r2 – x2) = 0  x=r 2
also f(r 2  ) < 0 and f(r 2  ) > 0
r
confirming at f(x) is maximum when x = r 2 & y = .
2
Aliter Let us choose coordinate system with origin as centre of circle (as shown in figure).
A = xy

 
 A = 2 (rcos) (rsin)  A = r2 sin2   0, 
 2
 r
Clearly A is maximum when  =  x=r 2 and y= .
4 2

Ex. A sheet of area 40 m2 is used to make an open tank with square base. Find the dimensions of
the base such that volume of this tank is maximum.
Sol. Let length of base be x meter and height be y meter (as shown in figure).
V = x2y
again x and y are related to surface area of
this tank which is equal to 40 m2.
 x2 + 4xy = 40
40  x 2
y= x  (0, 40 )
4x
 40  x 2 
 V(x) = x  4 x
2 

 
( 40 x  x 3 )
V(x) =
4
maximizing volume,
( 40  3x 2 ) 40
V(x) = =0  x= m
4 3

3x  40 
and V(x) = –  V  3  < 0.
2  

40
Confirming that volume is maximum at x = m.
3

Ex. If a right circular cylinder is inscribed in a given cone. Find the dimensions of the cylinder
such that its volume is maximum.
Sol. Let x be the radius of cylinder and y be its height v = x2y
x, y can be related by using similar triangles (as shown in figure).

Campus-I : F-106, Road No.2, Indraprastha Industrial Area, Kota Ph : 2420151 / 52, 9166876626
Campus-II : A-10, Jawahar Nagar Main Road, Kota Ph : 2406123 / 24
23
Mathematics (JEE-Advanced) Application of Derivative
y h
=
rx r

h
 y= (r – x)
r

h
 v(x) = x2 (r – x) x  (0, r)
r

h
 v(x) = (rx2 – x3)
r

h
v(x) = x (2r – 3x)
r
 2r   2r 
v  3  = 0 and v  3  < 0
   
 2r  h
Thus volume is maximum at x =  3  and y = .
  3
Note : Following formulae of volume, surface area of important solids are very useful in problems of
maxima & minima.
Ex. Among all regular square pyramids of volume 36 2 cm3. Find dimensions of the pyramid having
least lateral surface area.
Sol. Let the length of a side of base be x cm and y be the perpendicular height of the pyramid
(see figure).
1
V = 3 × area of base x height
1
 V= x2y = 36 2
3

108 2
 y=
x2
1
and S= × perimeter of base x slant height
2
1
= (4x). 
2
x2
but =  y2
4
2
x2 4
108 2  8.(108 )2
 S = 2x  y2 = x 4  4x 2 y 2 S= x  4x 2  S(x) = x 4 
4  2
x  x2
 
8.(108 )2
Let f(x) = x4 + for minimizing f(x)
x2
16(108 )2 (x 6  66 )
f(x) = 4x3 – = 0  f(x) = 4 = 0  x = 6, which a point of minima
x3 x3
Hence x = 6 cm and y = 3 2 .

Ex. Let A(1, 2) and B(– 2, – 4) be two fixed points. A variable point P is chosen on the straight line
y = x such that perimeter of PAB is minimum. Find coordinates of P.

Campus-I : F-106, Road No.2, Indraprastha Industrial Area, Kota Ph : 2420151 / 52, 9166876626
Campus-II : A-10, Jawahar Nagar Main Road, Kota Ph : 2406123 / 24
24
Mathematics (JEE-Advanced) Application of Derivative
Sol. Since distance AB is fixed so for minimizing the perimeter
of PAB, we basically have to minimize (PA + PB)
Let A be the mirror image of A in the line y = x (see figure).
F(P) = PA + PB
F(P) = PA + PB
But for PAB
PA + PB  AB and equality hold when P, A and B becomes collinear. Thus for minimum path
length point P is that special point for which PA and PB become incident and reflected rays
with respect to the mirror y = x.
Equation of line joining A and B is y = 2x intersection of this line with y = x is the point P.
Hence P  (0, 0).

Ex. If 0  c  5, then the minimum distance of the point (0,c) from parabola y = x2 is-
(A) c  4 (B) c  1 / 4 (C) c  1 / 4 (D) None of these
Sol. Let e t, t j be a point on the parabola whose distance from (0,c), be d. Then
dz d2z
z = d2 = t + (t–c)2 = t2 + t(1–2c)+ c2  dt = 2t + 1 – 2c, = 2 > 0
d t2
dz
Now d t = 0  t = c – 1/2
which gives the minimum distance. So
min. distance = b c  1 / 2g  b  1 / 2g 2
= c  1/ 4
Ex. The maximum area of a rectangle of perimeter 176 cms. is-
(A) 1936 sq.cms. (B) 1854 sq.cms. (C) 2110 sq.cms. (D) None of these
Sol. Let sides of the rectangle be x, y ; then
2x + 2y = 176 ...(1)
 Its area A = xy = x (88– x) [form (1)] = 88x – x2
dA d2 A
 = 88 – 2x, = – 2 < 0
dx dx2
dA d2 A
Now = 0  x = 44; Also then < 0. So area will be maximum when x = 44 and
dx dx2
maximum area
= 44 x 44 = 1936 sq. cms.

Ex. The semivertical angle of a right circular cone of given slant height and maximum volume is-

e 2j FG 1IJ e j
(A) tan–1 2 (B) tan–1 (C) tan–1 2 (D) tan–1 1 / 2 H K
Sol. Let  be the slant height and  be the semivertical angle of the right circular cone. Also
suppose that h and r are its height and radius of the base.
Campus-I : F-106, Road No.2, Indraprastha Industrial Area, Kota Ph : 2420151 / 52, 9166876626
Campus-II : A-10, Jawahar Nagar Main Road, Kota Ph : 2406123 / 24
25
Mathematics (JEE-Advanced) Application of Derivative
Then h =  cos , r =  sin
1
Now volume V =  r2 h
3
1
= 3 sin2  cos 
3
dV 1
 =   3 [–sin3  + 2 sin  cos2 ]
d 3
1 1
=   3[ – sin3  + 2 sin  (1 – sin2 )] =   3[ 2 sin  – 3 sin3 ]
3 3
d2 V 1
 =   3[2 cos  – 9 sin2  cos ]
d 2 3
dV
Now = 0  sin  = 0 or 2–3 sin2 = 0
d
Now   0  2 = 3 sin2
or 2 sin2  + 2 cos2  = 3 sin2 
or tan2  = 2  tan  = 2
d2 V
When tan  = 2, < 0
d 2
Thus when  = tan–1 2 , volume
will be maximum.
7. SIGNIFICANCE OF THE SIGN OF 2ND ORDER DERIVATIVE AND POINTS OF INFLECTION :
The sign of the 2nd order derivative determines the concavity
of the curve. Such points such as C & E on the graph where
the concavity of the curve changes are called the points of
inflection. From the graph we find that if:
d2y
(i) > 0  concave upwards
dx 2

d2y
(ii) < 0  concave downwards.
dx 2

d2y
At the point of inflection we find that =0&
dx 2

d2y
changes sign.
dx 2

d2y
Inflection points can also occur if 2 fails to exist. For example, consider the graph of the function
dx
defined as,
x 3/ 5 for x  (  , 1)
f (x) = [ 2  x 2 for x  (1 , )
Note that the graph exhibits two critical points one is a point of local
maximum & the other a point of inflection.

Campus-I : F-106, Road No.2, Indraprastha Industrial Area, Kota Ph : 2420151 / 52, 9166876626
Campus-II : A-10, Jawahar Nagar Main Road, Kota Ph : 2406123 / 24
26
Mathematics (JEE-Advanced) Application of Derivative
EXERCISE - I
Rate of change, Approximation
Q.1 On the curve x3 = 12y. The interval in which abscissa changes at a faster rate then its ordinate
(A) (0, 2) (B) (–, –2)  (2, ) (C) (–2, 2) (D) none of these

Q.2 Using differentials, find the approximate value of 25.2 .

(A) 5.02 (B) 5.01 (C) 5.03 (D) 5.04


Q.3 The approximate change in the volume of a cube of side x meters caused by increasing the side by 4%
is
(A) 0.06x3m3 (B) 0.09x3m3 (C) 0.12x3m3 (D) 0.15x3m3
Tangent, Normal, Angle between curves, Orthogonality of Curves, Shortest distance, Length of
Tangent & Normal
Q.4 The angle at which the curve y = KeKx intersects the y-axis is :

(A) tan1 k2 (B) cot1 (k2) (C) sec1  1  k 4  (D) none


 

Q.5 The curve y  exy + x = 0 has a vertical tangent at


(A) (1, 1) (B) (0, 1) (C) (1, 0) (D) no point
Q.6 The angle made by the tangent of the curve x = a (t + sint cost) ; y = a (1 + sint)2 with the x-axis at any
point on it is

1 1 sin t 1 1 sin t
(A)   2 t  (B) (C) 2 t   (D)
4 cos t 4 cos 2 t
Q.7 Number of tangents drawn from the point (–1/2, 0) to the curve y = e{x}. (Here { } denotes fractional
part function).
(A) 2 (B) 1 (C) 3 (D) 4
7
Q.8 The tangent to y = ax2 + bx + at (1, 2) is parallel to the normal at the point (–2, 2) on the curve
2
y = x2 + 6x + 10, then the value of a and b equals
(A) (–1, –5/2) (B) (1, –5/2) (C) (1, 5/2) (D) (–1, 5/2)
Q.9 The equation of the normal to the curve y = (1 + x)y + sin1 (sin2x) at x = 0 is
(A) x + y – 1 = 0 (B) x + y + 1 = 0 (C) 2x + y – 1 = 0 (D) x + 2y – 1 = 0
Q.10 The graphs y = 2x3 – 4x + 2 and y = x3 + 2x – 1 intersect in exactly 3 distinct points. The slope of the
line passing through two of these points
(A) is equal to 4 (B) is equal to 6 (C) is equal to 8 (D) is not unique
Q.11 If curve y = 1 – ax2 and y = x2 intersect orthogonally then the value of a is
(A) 1/2 (B) 1/3 (C) 2 (D) 3
Q.12 Let C be the curve y = x3 (where x takes all real values). The tangent at A meets the curve again at B.
If the gradient at B is K times the gradient at A then K is equal to
(A) 4 (B) 2 (C) – 2 (D) 1/4

Campus-I : F-106, Road No.2, Indraprastha Industrial Area, Kota Ph : 2420151 / 52, 9166876626
Campus-II : A-10, Jawahar Nagar Main Road, Kota Ph : 2406123 / 24
27
Mathematics (JEE-Advanced) Application of Derivative
2
Q.13 The coordinates of the point of the parabola y = 8x , which is at minimum distance from the circle
x2 + (y + 6)2 = 1 are
(A) (2 ,  4) (B) (18 , 12) (C) (2 , 4) (D) none of these
Q.14 A curve is represented by the equations, x = sec2 t and y = cot t where t is a parameter. If the tangent
at the point P on the curve where t = /4 meets the curve again at the point Q then PQ is equal to:

5 3 5 5 2 5 3 5
(A) (B) (C) (D)
2 2 3 2
Q.15 A curve with equation of the form y = ax4 + bx3 + cx + d has zero gradient at the point (0, 1) and also
touches the x  axis at the point ( 1, 0) then the values of x for which the curve has a negative
gradient are:
(A) x >  1 (B) x < 1 (C) x <  1 (D)  1  x  1
Monotonicity on an interval, Monotonicity about a point

1 x2 
Q.16 The function f (x) = tan–1  
 1  x 2  is
 
(A) increasing in its domain
(B) decreasing in its domain
(C) decreasing in (–, 0) and increasing in (0, )
(D) increasing in (– , 0) and decreasing in (0, )
Q.17 If f and g are two increasing functions such that gof is defined, then
(A) gof is an increasing function (B) gof is a decreasing function
(C) gof is neither increasing nor decreasing (D) None of these
Q.18 The function xx decreases on the interval
 1
(A) (0, e) (B) (0, 1) (C)  0, e  (D) None of these

Q.19 If f (x ) = xe x (1- x ) , then f (x) is

 1 
(A) increasing on   2 , 1 (B) decreasing R
 
 1 
(C) increasing on R (D) decreasing on  – , 1
 2 

 
Q.20 The function 2tan3x – 3tan2x + 12tanx + 3, x   0, 2  is
 

(A) increasing (B) decreasing


(C)increasing in (0, /4) and decreasing in (/4, /2) (D) none of these
Q.21 Function f(x) = (x – 1)2 (x – 2) is monotonically decreasing when –
(A) x  (1, 2) (B) x  (1, 5/3) (C) x  R – (1, 5/3) (D) No where

Campus-I : F-106, Road No.2, Indraprastha Industrial Area, Kota Ph : 2420151 / 52, 9166876626
Campus-II : A-10, Jawahar Nagar Main Road, Kota Ph : 2406123 / 24
28
Mathematics (JEE-Advanced) Application of Derivative
FG   IJ x
H K
Q.22 In the interval  3 , 3 , the function f(x) =
2
+ sin x is–

(A) Increasing (B) Decreasing


(C) Constant (D) Neither increasing nor decreasing

 x  1|
Q.23 The function is monotonically decreasing at the point
x2

(A) x = 3 (B) x = 1 (C) x = 2 (D) none of these


 p4 
Q.24 The values of p for which the function f(x) =   1 x5 – 3x + ln 5 decreases for all real x is

1 p  

 3  21 
(A) (– , ) (B)  4,   (1, )
 2 

 5  27 
(C)  3,   (2, ) (D) [1, )
 2 

Q.25 Which of the following statements is/are correct ?


(A) x + sinx is increasing function
(B) sec x is neither increasing nor decreasing function
(C) x + sinx is decreasing function
(D) sec x is an increasing function
Local maxima, Local minima, Global maxima, Global minima, Application of Maxima and Minima
Q.26 Difference between the greatest and the least values of the function f (x) = x(ln x – 2) on [1, e2] is
(A) 2 (B) e (C) e2 (D) 1
Q.27 If f(x) = 1 + 2 x2 + 4 x4 + 6 x6 +...... + 100 x100 is a polynomial in a real variable x, then f(x) has:
(A) neither a maximum nor a minimum (B) only one maximum
(C) only one minimum (D) one maximum and one minimum

Q.28 The greatest, the least values of the function, f(x) = 2 1 2x  x2 , x  [21] are respectively

(A) 2, 1 (B) 2,  1 (C) 2, 0 (D) none


Q.29 Let f (x) = x3 – 3x2 + 2x. If the equation f (x) = k has exactly one positive and one negative solution then
the value of k equals

2 3 2 2 1
(A) – (B) – (C) (D)
9 9 3 3 3 3
Q.30 If f(x) = a n |x| + bx2 + x has its extremum values at x = – 1 and x = 2, then
(A) a = 2, b = – 1 (B) a = 2, b = – 1/2 (C) a = – 2, b = 1/2 (D) none of these

Q.31 The minimum value of



tan x  6 
is :
tan x

(A) 0 (B) 1/2 (C) 1 (D) 3

Campus-I : F-106, Road No.2, Indraprastha Industrial Area, Kota Ph : 2420151 / 52, 9166876626
Campus-II : A-10, Jawahar Nagar Main Road, Kota Ph : 2406123 / 24
29
Mathematics (JEE-Advanced) Application of Derivative
Q.32 If a < b < c < d & x  R then the least value of the function,
f(x) = x  a + x  b + x  c + x  d is
(A) c – d + b – a (B) c + d – b – a (C) c + d – b + a (D) c – d + b + a
Q.33 Let f(x) = (1 + b2)x2 + 2bx + 1 and let m(b) be the minimum value of f(x). As b varies, the range
of m(b) is

 1  1 
(A) [0, 1] (B)  0, 2  (C)  2 , 1 (D) (0, 1]
   

Q.34 The radius of a right circular cylinder of greatest curved surface which can be inscribed in a given
right circular cone is

(A) one third that of the cone (B) 1/ 2 times that of the cone
(C) 2/3 that of the cone (D) 1/2 that of the cone
Q.35 The true set of real values of x for which the function, f(x) = x ln x  x + 1 is positive is
(A) (1, ) (B) (1/e, ) (C) [e, ) (D) (0, 1)  (1, )
Q.36 The dimensions of the rectangle of maximum area that can be inscribed in the ellipse
(x/4)2 + (y/3)2 = 1 are

(A) 8, 2 (B) 4, 3 (C) 2 8 , 3 2 (D) 2 , 6

x3 5 if x  1
Q.37 Let f (x) =  , then the number of critical points on the graph of the function is

 (x  2)3 if x  1

(A) 1 (B) 2 (C) 3 (D) 4

 x1
if x  1
Q.38 Let a function f be defined as f (x) =  x2  1
 x2 if x  1

Then the number of critical point(s) on the graph of this function is/are :
(A) 4 (B) 3 (C) 2 (D) 1
Q.39 The largest area of a rectangle which has one side on the xaxis and the two vertices on the curve
2
y = e  x is

(A) 2 e 1/2 (B) 2 e 1/2 (C) e 1/2 (D) none


Q.40 The maximum distance of the point (k, 0) from the curve 2x2 + y2 – 2x = 0 is equal to

(A) 1  2k  k 2 (B) 1  2k  2k 2 (C) 1  2k  2k 2 (D) 1  2k  k 2


Q.41 Which one of the following can best represent the graph of the function f (x) = 3x4 – 4x3?

(A) (B) (C) (D)

Campus-I : F-106, Road No.2, Indraprastha Industrial Area, Kota Ph : 2420151 / 52, 9166876626
Campus-II : A-10, Jawahar Nagar Main Road, Kota Ph : 2406123 / 24
30
Mathematics (JEE-Advanced) Application of Derivative
Q.42 The graph of y = f (x) is shown. Let F (x) be an antiderivative of f (x). Then F(x) has

2 4 
(A) points of inflexion at x = 0, , , and 2, a local maximum at x = , and a local minimum
3 3 2
3
at x =
2
2 4 
(B) points of inflexion at x = 0, , , and 2, a local minimum at x = , and a local maximum
3 3 2
3
at x =
2
 3
(C) point of inflexion at x = , a local maximum at x = , and a local minimum at x =
2 2
 3
(D) point of inflexion at x = , a local minimum at x = , and a local maximum at x =
2 2
Q.43 Two sides of a triangle are to have lengths 'a' cm & 'b' cm. If the triangle is to have the maximum area,
then the length of the median from the vertex containing the sides 'a' and 'b' is

1 2 2a  b a2  b2 a  2b
(A) a  b2 (B) (C) (D)
2 3 2 3
Q.44 Let f (x) = ax2 – b | x |, where a and b are constants. Then at x = 0, f (x) has
(A) a maxima whenever a > 0, b > 0
(B) a maxima whenever a > 0, b < 0
(C) minima whenever a > 0, b > 0
(D) neither a maxima nor minima whenever a > 0, b < 0
Q.45 The least value of 'a' for which the equation,

4 1
 = a has atleast one solution on the interval (0, /2) is :
sin x 1  sin x

(A) 3 (B) 5 (C) 7 (D) 9


Q.46 Number of critical points of the function,
x
2 x
f(x) =
3
x3  
2 
1
 12  12 cos2 t  t  dt
which lie in the interval [ 2, 2] is :
(A) 2 (B) 4 (C) 6 (D) 8
Campus-I : F-106, Road No.2, Indraprastha Industrial Area, Kota Ph : 2420151 / 52, 9166876626
Campus-II : A-10, Jawahar Nagar Main Road, Kota Ph : 2406123 / 24
31
Mathematics (JEE-Advanced) Application of Derivative
Curvature, Points of inflection, Inequalities
Q.47 If the function f (x) = x4 + bx2 + 8x + 1 has a horizontal tangent and a point of inflection for the same
value of x then the value of b is equal to
(A) – 1 (B) 1 (C) 6 (D) – 6
Q.48 The curve y = f(x) which satisfies the condition f  (x) > 0 and f  (x) < 0 for all real x, is:

(A) (B) (C) (D)

3x 2
Q.49 For which values of ‘a’ will the function f(x) = x4 + ax3 + + 1 will be concave upward along
2
the entire real line
(A) a  [0, ) (B) a  (–2, ) (C) a  [–2, 2] (D) a  (0, )
Q.50 If the point (1, 3) serves as the point of inflection of the curve y = ax3 + bx2 then the value of 'a'
and 'b' are:
(A) a = 3/2 & b =  9/2 (B) a = 3/2 & b = 9/2
(C) a =  3/2 & b =  9/2 (D) a =  3/2 & b = 9/2
Rolle's Theorem, LMVT

1 1 1
2 3
Q.51 Let f (x) = 3  x 5  3 x 3 x  1 . Then the equation f (x) = 0 has
2 5 8
2 x  1 3x  1 7 x  1
(A) no real root
(B) atmost one real root
(C) atleast 2 real roots
(D) exactly one real root in (0,1) and no other real root.

Q.52 In which of the following functions Rolle’s theorem is applicable?


sin x
x , 0 x  1  x ,    x  0

(A) f(x) =  on [0, 1] (B) f(x) =  on [–, 0]
0 , x 1  0 , x 0

x 3  2 x 2  5x  6
2
x x 6  if x  1, on [2,3]
(C) f(x) = on [–2,3] (D) f(x) =  x 1
x 1
  6 if x  1
3 2
Q.53 The function f(x) = x – 6x + ax + b satisfy the conditions of Rolle's theorem on [1, 3]. Which of
these are correct ?
(A) a =11, b  R (B) a = 11, b = – 6 (C) a = –11, b = 6 (D) a = – 11, b  R
Campus-I : F-106, Road No.2, Indraprastha Industrial Area, Kota Ph : 2420151 / 52, 9166876626
Campus-II : A-10, Jawahar Nagar Main Road, Kota Ph : 2406123 / 24
32
Mathematics (JEE-Advanced) Application of Derivative
Q.54 Consider f (x) = | 1 – x | 1  x  2 and

g (x) = f (x) + b sin x, 1<x<2
2
then which of the following is correct?
3
(A) Rolles theorem is applicable to both f, g and b =
2
1
(B) LMVT is not applicable to f and Rolles theorem if applicable to g with b =
2
(C) LMVT is applicable to f and Rolles theorem is applicable to g with b = 1
(D) Rolles theorem is not applicable to both f, g for any real b.
ta n [ x]
2/3  , x 0
1  x
Q.55 Given: f (x) = 4    x  g (x) = 
2   1 , x 0

h (x) = {x} k (x) = 5log 2 ( x  3)


then in [0, 1], Lagranges Mean Value Theorem is NOT applicable to
(A) f, g, h (B) h, k (C) f, g (D) g, h, k
where [x] and {x} denotes the greatest integer and fraction part function.

Q.56 The function f(x) = x(x + 3)e–x/2 satisfies all the conditions of Rolle's theorem on [–3, 0]. The value
of c which verifies Rolle's theorem, is
(A) 0 (B) – 1 (C) – 2 (D) 3

Q.57 Suppose that f is differentiable for all x and that f '(x)  2 for all x. If f (1) = 2 and f (4) = 8 then f (2)
has the value equal to
(A) 3 (B) 4 (C) 6 (D) 8

Campus-I : F-106, Road No.2, Indraprastha Industrial Area, Kota Ph : 2420151 / 52, 9166876626
Campus-II : A-10, Jawahar Nagar Main Road, Kota Ph : 2406123 / 24
33
Mathematics (JEE-Advanced) Application of Derivative
EXERCISE - II(A)
(TANGENT & NORMAL)
Q.1 Equation of a line which is tangent to both the curves y = x2 + 1 and y = – x2 is
1 1
(A) y = 2x  (B) y = 2x 
2 2
1 1
(C) y = – 2x  (D) y = – 2x 
2 2
x y K
Q.2 If  = 1 is a tangent to the curve x = Kt, y = , K > 0 then :
a b t
(A) a > 0, b > 0 (B) a > 0, b < 0 (C) a < 0, b > 0 (D) a < 0, b < 0
x 3 5x 2
Q.3 The co-ordinates of the point(s) on the graph of the function, f(x) =  + 7x - 4 where the
3 2
tangent drawn cut off intercepts from the co-ordinate axes which are equal in magnitude but opposite
in sign, is
(A) (2, 8/3) (B) (3, 7/2) (C) (1, 5/6) (D) None of these
Q.4 If tangent to curve 2y3 = ax2 + x3 at point (a, a) cuts off intercepts ,  on co-ordinate axes,
where 2 +  2 = 61, then the value of 'a' is equal to
(A) 20 (B) 25 (C) 30 (D)  30
x 3 5x 2
Q.5 The co-ordinates of point(s) on the graph of the function, f(x) =  + 7x – 4 where the
3 2
tangent drawn cut off intercepts from the co-ordinate axes which are equal in magnitude but
opposite in sign, is
(A) (2, 8/3) (B) (3, 7/2) (C) (1, 5/6) (D) none

(MONOTONOCITY)
 2 
Q.1 If f(x) = 2x + cot –1 x + n  1  x  x  , then f(x) :
(A) increases in [0, ) (B) decreases in [0, )
(C) neither increases nor decreases in [0, ) (D) increases in (– , )

Q.2 If f(x) = a
a x
sgn x  ; g(x) = a a x
sgn x 

for a > 0, a  1 and x  R, where { } & [ ] denote the
fractional part and integral part functions respectively, then which of the following statements can hold
good for the function h(x), where
(ln a) h(x) = (ln f(x) + ln g(x)).
(A) ‘h’ is even and increasing (B) ‘h’ is odd and decreasing
(C) ‘h’ is even and decreasing (D) ‘h’ is odd and increasing.
Q.3 On which of the following intervals, the function x100 + sin x  1 is strictly increasing.
(A) ( 1, 1) (B) (0, 1) (C) (/2, ) (D) (0, /2)
3x 2  12 x  1 ,  1  x  2
Q.4 If f (x) =  then :
 37  x , 2 x  3

(A) f(x) is increasing on [ 1 , 2] (B) f(x) is continuous on [ 1 , 3]


(C) f  (2) does not exist (D) f(x) has the maximum value at x = 2 .
Campus-I : F-106, Road No.2, Indraprastha Industrial Area, Kota Ph : 2420151 / 52, 9166876626
Campus-II : A-10, Jawahar Nagar Main Road, Kota Ph : 2406123 / 24
34
Mathematics (JEE-Advanced) Application of Derivative
Q.5 Consider the function f (x) = x2 – x sin x – cos x then the statements which holds good, are
(A) f (x) = k has no solution for k < – 1. (B) f is increasing for x < 0 and decreasing for x > 0.

(C) xLim

f (x )   (D) The zeros of f (x) = 0 lie on the same side of the origin.
Q.6 Assume that inverse of the function f is denoted by g. Then which of the following statement hold
good?
(A) If f is increasing then g is also increasing. (B) If f is decreasing then g is increasing.
(C) The function f is injective. (D) The function g is onto.
Q.7 For the function f (x) = ln (1 – ln x) which of the following do not hold good?
(A) increasing in (0, 1) and decreasing in (1, e) (B) decreasing in (0, 1) and increasing in (1, e)
(C) x = 1 is the critical number for f (x). (D) f has two asymptotes

[Comprehension Type]
Paragraph for question nos. 8 to 10
Consider the cubic f (x) = 8x3 + 4ax2 + 2bx + a where a, b  R.
Q.8 For a = 1 if y = f (x) is strictly increasing  x  R then maximum range of values of b is

 1 1  1 
(A)   ,  (B)  ,   (C)  ,   (D) (–, )
 3 3  3 
Q.9 For b = 1, if y = f (x) is non monotonic then the sum of all the integral values of a  [1, 100], is
(A) 4950 (B) 5049 (C) 5050 (D) 5047
Q.10 If the sum of the base 2 logarithms of the roots of the cubic f (x) = 0 is 5 then the value of 'a' is
(A) – 64 (B) – 8 (C) – 128 (D) – 256
[Matrix Match]
Q.11 Column I contains function and Column II contains behaviours of function in their domain. Entry of
column I are to be matched with one or more than one entries of column II.
Column-I Column-II

(A) f (x) = 5 (P) monotonic


x 5

ln x
(B) g (x) = (Q) non monotonic
x
(C) h (x) = x sgn x, where sgn x denotes signum function of x. (R) possesses extremum point

 tan 1 x , if x  0

(D) k (x) =  2 , if x  0 (S) possesses critical point
 cot 1 x , if x  0

(T) possesses inflection point.

Campus-I : F-106, Road No.2, Indraprastha Industrial Area, Kota Ph : 2420151 / 52, 9166876626
Campus-II : A-10, Jawahar Nagar Main Road, Kota Ph : 2406123 / 24
35
Mathematics (JEE-Advanced) Application of Derivative
(MAXIMA & MINIMA)
[MULTIPLE CORRECT QUESTION]
Q.1 If f(x) = tan–1x – (1/2) n x. Then

 
(A) the greatest value of f(x) on 1/ 3 , 3 is /6 + (1/4) n 3

 
(B) the least value of f(x) on 1/ 3 , 3 is /3 – (1/4) n 3

(C) f(x) decreases on (0, )


(D) f(x) increases on (– , 0)
Q.2 Let f(x) = (x2  1)n (x2 + x + 1). f(x) has local extremum at x = 1 if
(A) n = 2 (B) n = 3 (C) n = 4 (D) n = 6
x  
Q.3 If f(x) = 1  x tan x , x   0,  , then
 2

(A) f(x) has exactly one point of minima (B) f(x) has exactly one point of maxima

(C) f(x) is increasing in  0,  (D) maxima occurs at x0 where x0 = cosx0
 2
x 1
Q.4 Let f (x) = then which of the following is correct.
x2
(A) f (x) has minima but no maxima.
(B) f (x) increases in the interval (0, 2) and decreases in the interval (– , 0)  (2, ).
(C) f (x) is concave down in (– , 0)  (0, 3).
(D) x = 3 is the point of inflection.
Q.5 The function f (x) = x1/3(x – 1)
(A) has 2 inflection points.
(B) is strictly increasing for x > 1/4 and strictly decreasing for x < 1/4.
(C) is concave down in (– 1/2, 0).
(D) Area enclosed by the curve lying in the fourth quadrant is 9 28 .
x
Q.6 The function f(x) =  1  t 4 dt is such that :
0

(A) it is defined on the interval [ 1, 1] (B) it is an increasing function


(C) it is an odd function (D) the point (0, 0) is the point of inflection

Q.7 A function f is defined by f (x) =  cos t cos(x  t )dt , 0  x  2 then which of the following hold(s) good?
0

(A) f (x) is continuous but not differentiable in (0, 2)


(B) Maximum value of f is 
(C) There exists atleast one c  (0, 2) s.t. f ' (c) = 0.

(D) Minimum value of f is – .
2
Campus-I : F-106, Road No.2, Indraprastha Industrial Area, Kota Ph : 2420151 / 52, 9166876626
Campus-II : A-10, Jawahar Nagar Main Road, Kota Ph : 2406123 / 24
36
Mathematics (JEE-Advanced) Application of Derivative
x2 if x  1
 2
Q.8 The function f (x) =  x if  1  x  1
 ( x  2) 2 if x  1

(A) is continuous for all x  R


(B) is continuous but not differentiable  x  R
(C) is such that f ' (x) change its sign exactly twice
(D) has two local maxima and two local minima.

Paragraph for question nos. 9 to 11

x2
Consider the function f (x) =
x 2 1
Q.9 The interval in which f is increasing is
(A) (–1, 1) (B) (– , – 1)(–1, 0)
(C) (– , ) – {–1, 1} (C) (0, 1)  (1, )
Q.10 If f is defined from R – {–1, 1}  R then f is
(A) injective but not surjective (B) surjective but not injective
(C) injective as well as surjective (D) neither injective nor surjective.
Q.11 f has
(A) local maxima but no local minima (B) local minima but no local maxima
(C) both local maxima and local minima (D) neither local maxima nor local minima.
Paragraph for question nos. 12 to 14

 x ln 1  (1 x ) if 0  x  1
x
 1
Let f (x) = 1   (x > 0) and g (x) = 
 x  0 if x  0

Q.12 Lim g ( x )
x 0 

(A) is equal to 0 (B) is equal to 1 (C) is equal to e (D) is non existent


Q.13 The function f
(A) has a maxima but no minima (B) has a minima but no maxima
(C) has both a maxima and minima (D) is monotonic
1n
 1  2  3  n 
Q.14 Lim  f  · f  · f   ......... f   equals
n   n n n  n 

(A) 2e (B) 2e (C) 2 e (D) e

Campus-I : F-106, Road No.2, Indraprastha Industrial Area, Kota Ph : 2420151 / 52, 9166876626
Campus-II : A-10, Jawahar Nagar Main Road, Kota Ph : 2406123 / 24
37
Mathematics (JEE-Advanced) Application of Derivative
Paragraph for question nos. 15 to 17
Suppose you do not know the function f (x), however some information about f (x) is listed below.
Read the following carefully before attempting the questions
(i) f (x) is continuous and defined for all real numbers
(ii) f '(–5) = 0 ; f '(2) is not defined and f '(4) = 0
(iii) (–5, 12) is a point which lies on the graph of f (x)
(iv) f ''(2) is undefined, but f ''(x) is negative everywhere else.
(v) the signs of f '(x) is given below

Q.15 On the possible graph of y = f (x) we have


(A) x = – 5 is a point of relative minima.
(B) x = 2 is a point of relative maxima.
(C) x = 4 is a point of relative minima.
(D) graph of y = f (x) must have a geometrical sharp corner.
Q.16 From the possible graph of y = f (x), we can say that
(A) There is exactly one point of inflection on the curve.
(B) f (x) increases on – 5 < x < 2 and x > 4 and decreases on –  < x < – 5 and 2 < x < 4.
(C) The curve is always concave down.
(D) Curve always concave up.
Q.17 Possible graph of y = f (x) is

(A) (B)

(C) (D)

Campus-I : F-106, Road No.2, Indraprastha Industrial Area, Kota Ph : 2420151 / 52, 9166876626
Campus-II : A-10, Jawahar Nagar Main Road, Kota Ph : 2406123 / 24
38
Mathematics (JEE-Advanced) Application of Derivative

EXERCISE - II(B)
(TANGENT & NORMAL, ROLLE'S, LMVT AND RATE MEASURE)
Q.1 Find the point of intersection of the tangents drawn to the curve x2y = 1 – y at the points where it is
intersected by the curve xy = 1 – y.

Q.2 A straight line is drawn through the origin and parallel to the tangent to a curve

 a  a2  y2 
x  a 2  y2
= ln  
a y  at an arbitary point M. Show that the locus of the point P of
 
intersection of the straight line through the origin & the straight line parallel to the x-axis & passing
through the point M is x2 + y2 = a2.
41x 3
Q.3 A line is tangent to the curve f (x) = at the point P in the first quadrant, and has a slope of 2009.
3
This line intersects the y-axis at (0, b). Find the value of 'b'.

Q.4 A function is defined parametrically by the equations

1 1
 2t  t 2 sin if t  0  sin t 2 if t  0
f(t) = x =  t and g(t) = y =  t
 0 if t  0  o if t  0
Find the equation of the tangent and normal at the point for t = 0 if exist.

Q.5 Find all the tangents to the curve y = cos (x + y),  2  x  2, that are parallel to the line x + 2y = 0.

Q.6 Find the segment of the normal to the curve x = 2a sin t + a sin t cos2t ; y =  a cos3t contained
between the co-ordinate axes.
8
Q.7 There is a point (p, q) on the graph of f (x) = x2 and a point (r, s) on the graph of g (x) =
x
where p > 0 and r > 0. If the line through (p, q) and (r, s) is also tangent to both the curves at these
points respectively, then find the value of (p + r).

Q.8 If the tangent at the point (x1, y1) to the curve x3 + y3 = a3 (a  0) meets the curve again in (x2, y2) then
x 2 y2
show that  =  1.
x1 y1
Q.9 Determine a differentiable function y = f (x) which satisfies f ' (x) = [f(x)]2 and f (0) = – 1/2. Find also
the equation of the tangent at the point where the curve crosses the y-axis.
Q.10 Tangent at a point P1 [other than (0 , 0)] on the curve y = x3 meets the curve again at P2. The tangent
at P2 meets the curve at P3 & so on. Show that the abscissae of P1, P2, P3, ......... Pn, form a GP. Also
area ( P1 P2 P3 )
find the ratio .
area ( P2 P3 P4 )
Q.11 The curve y = ax3 + bx2 + cx + 5 , touches the x - axis at P ( 2 , 0) & cuts the y-axis at a point Q where
its gradient is 3. Find a , b , c.
Q.12 Find the condition that the curves x2/3 + y2/3 = c2/3 & (x2/a2) + (y2/b2) = 1 may touch if c = a + b.
Q.13 The graph of a certain function f contains the point (0, 2) and has the property that for each number 'p'
the line tangent to y = f (x) at p, f (p)  intersect the x-axis at p + 2. Find f (x).
Campus-I : F-106, Road No.2, Indraprastha Industrial Area, Kota Ph : 2420151 / 52, 9166876626
Campus-II : A-10, Jawahar Nagar Main Road, Kota Ph : 2406123 / 24
39
Mathematics (JEE-Advanced) Application of Derivative
Q.14 L et C be the curve f (x) = ln2x + 2 ln x and Aa, f (a )  , Bb, f (b) where (a < b) are the points
of tangency of two tangents drawn from origin to the curve C.
(i) Find the value of the product ab.
(ii) Find the number of values of x satisfying the equation 5x f ' (x) – x ln 10 – 10 = 0.
y
Q.15 Show that the angle between the tangent at any point 'A' of the curve ln (x2 + y2) = C tan–1 and the
x
line joining A to the origin is independent of the position of A on the curve.
Q.16 A man 1.5 m tall walks away from a lamp post 4.5 m high at the rate of 4 km/hr.
How fast is the farther end of the shadow moving on the pavement ?
Q.17 Water is dripping out from a conical funnel of semi vertical angle /4, at the uniform rate of 2 cm3/sec
through a tiny hole at the vertex at the bottom. When the slant height of the water is 4 cm, find the rate
of decrease of the slant height of the water.
Q.18 A circular ink blot grows at the rate of 2 cm2 per second. Find the rate at which the radius is increasing
6 22
after 2 seconds. Use  = .
11 7
Q.19 Prove the inequality ex > (1 + x) using LMVT for all x  R0 and use it to determine which of the two
numbers e and e is greater.
Q.20 Verify Rolles throrem for f(x) = (x  a)m (x  b)n on [a, b] ; m, n being positive integer.
Q.21 Let f (x) = 4x3  3x2  2x + 1, use Rolle's theorem to prove that there exist c, 0< c <1 such that
f(c) = 0.
Q.22 Let f and g be functions, continuous in [a, b] and differentiable on [a, b]. If f (a) = f(b) = 0 then show
that there is a point c  (a, b) such that g ' (c) f (c) + f '(c) = 0.
Q.23 f (x) and g (x) are differentiable functions for 0  x  2 such that f (0) = 5, g (0) = 0, f (2) = 8, g (2) =
1. Show that there exists a number c satisfying 0 < c < 2 and f ' (c) = 3 g' (c).
3 x 0
 2
Q.24 For what value of a, m and b does the function f (x) =   x  3x  a 0  x  1
 mx  b 1 x  2
satisfy the hypothesis of the mean value theorem for the interval [0, 2].
Q.25 Let f be continuous on [a, b] and differentiable on (a, b). If f (a) = a and f (b) = b, show that there exist
distinct c1, c2 in (a, b) such that f ' (c1) + f '(c2) = 2.
Q.26 Let f defined on [0, 1] be a twice differentiable function such that, | f " (x) |  1 for all x  [0, 1]
If f (0) = f (1), then show that, | f ' (x) | < 1 for all x  [0, 1]
Q.27 Let f be a twice differentiable function on [0, 2] such that f (0) = 0, f (1) = 2, f (2) = 4, then prove
that
(a) f '() = 2 for some   (0, 1) (b) f '() = 2 for some   (1, 2)
(c) f "() = 0 for some   (0, 2)

Q.28 If f is a continuous function on the interval [a, b] and there exists some c  (a, b) then prove that

 f ( x ) dx = f (c) (b – a).
a

Campus-I : F-106, Road No.2, Indraprastha Industrial Area, Kota Ph : 2420151 / 52, 9166876626
Campus-II : A-10, Jawahar Nagar Main Road, Kota Ph : 2406123 / 24
40
Mathematics (JEE-Advanced) Application of Derivative
[MONOTONOCITY]
Q.1 Find the intervals of monotonocity for the following functions & represent your solution set on the number
line.
2
(a) f(x) = 2. e x 4x
(b) f(x) = ex/x (c) f(x) = x2 ex (d) f (x) = 2x2 – ln | x |
Also plot the graphs in each case & state their range.

Q.2 Let f (x) = 1 – x – x3. Find all real values of x satisfying the inequality, 1 – f (x) – f 3(x) > f (1 – 5x)

Q.3 Find the intervals of monotonocity of the functions in [0, 2]


(a) f (x) = sin x – cos x in x [0 , 2 ] (b) g (x) = 2 sinx + cos 2x in (0  x  2 ).
4 sin x  2 x  x cos x
(c) f (x) =
2  cos x
Q.4 Let f (x) be a increasing function defined on (0, ). If f (2a2 + a + 1) > f (3a2 – 4a + 1). Find the range of a.

max {f (t ) : 0  t  x} , 0  x  1
Q.5 Let f (x) = x3  x2 + x + 1 and g(x) = 
 3x ,1  x  2
Discuss the conti. & differentiability of g(x) in the interval (0,2).

Q.6 (a) Find the set of all values of the parameter 'a' for which the function,
f(x) = sin 2x – 8(a + 1)sin x + (4a2 + 8a – 14)x increases for all x  R and has no critical points for all x  R.
(b) Find all the values of the parameter 'a' for which the function ;
f(x) = 8ax  a sin 6x  7x  sin 5x increases & has no critical points for all x  R.
(c) Find the set of values of 'a' for which x the function f (x) = (a2 + a – 6)cos 2x + (a – 2)x + cos 1
has no critical points.
Q.7 Find the greatest & the least values of the following functions in the given interval if they exist.
x  1 
(a) f (x) = sin1  ln x in  , 3 (b) f (x) = 12x4/3 – 6x1/3, x  [–1, 1]
2
x 1  3 
(c) y = x5 – 5x4 + 5x3 + 1 in [ 1, 2]
1
Q.8 Find the values of 'a' for which the function f(x) = sin x  a sin2x  sin3x + 2ax increases throughout
3
the number line.
3 3
 1
Q.9 Find the minimum value of the function f (x) = x 2  x 2
 4  x   for all permissible real x.
 x
Q.10 If b > a, find the minimum value of (x  a)3+ (x  b)3, x  R.

Q.11 Find the set of values of 'a' for which the function,
 21  4 a  a 2  3
f(x) =  1  x + 5x + 7 is increasing at every point of its domain.
a 1 
 
Campus-I : F-106, Road No.2, Indraprastha Industrial Area, Kota Ph : 2420151 / 52, 9166876626
Campus-II : A-10, Jawahar Nagar Main Road, Kota Ph : 2406123 / 24
41
Mathematics (JEE-Advanced) Application of Derivative
Q.12 Find the value of x > 1 for which the function

x2
1  t  1  is increasing and decreasing.
F (x) =  t ln   dt
x
32 

Q.13 If f (x) = 2ex – ae–x + (2a + 1)x  3 monotonically increases for every x  R then find the range of
values of ‘a’.

Q.14 Prove that, x2 – 1 > 2x ln x > 4(x – 1) – 2 ln x for x > 1.

 3 
Q.15 Prove that tan2x + 6 ln secx + 2cos x + 4 > 6 sec x for x   , 2 .
 2 

x2
Q.16 Suppose that the function f (x) = log c is defined for all x in the interval [a, b], is monotonic
x2
decreasing. Find the value of 'c' for which there exists 'a' and 'b' (b> a> 2) such that the range of the
function is [logcc(b–1), logcc(a–1)].

Q.17 (a) Let f, g be differentiable on R and suppose that f (0) = g (0) and f ' (x)  g ' (x) for all x  0. Show
that f (x)  g (x) for all x  0.
(b) Show that exactly two real values of x satisfy the equation x2 = x sinx + cos x.

[Maxima & Minima]

Q.1 A cubic f(x) vanishes at x = 2 & has relative minimum/maximum at x = 1 and x = 1/3.
1
14
If  f ( x ) dx = 3
, find the cubic f (x).
1
x
Q.2 Investigate for maxima & minima for the function, f (x) =  [2 (t  1) (t  2)3 + 3 (t  1)2 (t  2)2] dt
1
Q.3 Find the greatest & least value for the function ;
(a) y = x + sin 2x , 0  x  2 (b) y = 2 cos 2x  cos 4x , 0  x  

Q.4 Suppose f(x) is a function satisfying the following conditions :


5
(i) f(0) = 2, f(1) = 1 (ii) f has a minimum value at x = and
2

2ax 2ax  1 2ax  b  1


(iii) for all x, f  (x) = b b 1 1
2(ax  b) 2ax  2b  1 2ax  b

Where a, b are some constants. Determine the constants a, b & the function f(x).

Q.5 Let P(x) be a polynomial of degree 5 having extremum at x = –1, 1 and Lim  P(x ) 
x 0  3
 2  4 .
 x 
If M and m are the maximum and minimum value of the function y = P ' (x) on the set
m
2

A = x x  6  5x then find  M
.
Campus-I : F-106, Road No.2, Indraprastha Industrial Area, Kota Ph : 2420151 / 52, 9166876626
Campus-II : A-10, Jawahar Nagar Main Road, Kota Ph : 2406123 / 24
42
Mathematics (JEE-Advanced) Application of Derivative

ax  b
Q.6 If y = has a turning value at (2, 1) find a & b and show that the turning value is a
(x  1) (x  4)
maximum.
6
Q.7 Of all the lines tangent to the graph of the curve y = 2 , find the equations of the tangent lines of
x 3
minimum and maximum slope.
N
n
Q.8 Let fN(x) be a polynomial function which satisfies f 'N (x) =  x  n  . If n1 is sum of
n 1

all values of x for which f10 (x) attains maxima and n2 denotes the sum of all values of x
for which f10 (x) attains minima, then find the value of (n1 + n2).
Q.9 Let f(x) be a polynomial of degree 6 divisible by x3, and having a point of extremum at x = 2.
3 f (2)
If f ' (x) is divisible by 1 + x2, then find the vlaue of .
f (1)

Q.10 Let P(x) be a polynomial of degree atmost four. It has zeros and also minima at x1 = – 3
and x2 = 5. Given P(x) has a local maximum value 256, then find the value of definite integral
1

 P(x )  P( x )  dx .
0

Q.11 Find the maximum perimeter of a triangle on a given base ‘a’ and having the given vertical angle.

Q.12 The length of three sides of a trapezium are equal, each being 10 cms. Find the maximum area of such
a trapezium.
x2 y 2
Q.13 A perpendicular is drawn from the centre to a tangent to an ellipse + = 1. Find the greatest
a2 b2
value of the intercept between the point of contact and the foot of the perpendicular.

Q.14 Given two points A ( 2 , 0) & B (0 , 4) and a line y = x. Find the co-ordinates of a point M on this line
so that the perimeter of the  AMB is least.
Q.15 A given quantity of metal is to be casted into a half cylinder i.e. with a rectangular base and semicircular
ends. Show that in order that total surface area may be minimum , the ratio of the height of the cylinder
to the diameter of the semi circular ends is /(+ 2).
1
Q.16 Let ,  be real numbers with 0     and f (x) = x2 – ( + )x +  such that  f ( x ) dx = 1. Find
 1
the maximum value of  f ( x ) dx .
0

Q.17 Show that for each a > 0 the function eax. xa² has a maximum value say F (a), and that F (x) has a
minimum value, ee/2.
Q.18 Let f (x) be a cubic polynomial which has local maximum at x = – 1 and f '(x) has a local minimum
at x = 1. If f(–1) = 10 and f(3) = – 22, then find the distance between its two horizontal tangents.

Campus-I : F-106, Road No.2, Indraprastha Industrial Area, Kota Ph : 2420151 / 52, 9166876626
Campus-II : A-10, Jawahar Nagar Main Road, Kota Ph : 2406123 / 24
43
Mathematics (JEE-Advanced) Application of Derivative
x ln x when x  0

Q.19 Consider the function f (x) = 
 0 for x  0
(a) Find whether f is continuous at x = 0 or not.
(b) Find the minima and maxima if they exist.
(c) Does f ' (0) ? Find Lim f ' ( x ) .
x 0
(d) Find the inflection points of the graph of y = f (x)..
Q.20 Consider the function y = f (x) = ln (1 + sin x) with – 2  x  2. Find
(a) the zeroes of f (x)
(b) inflection points if any on the graph
(c) local maxima and minima of f (x)
(d) asymptotes of the graph 2
(e) sketch the graph of f (x) and compute the value of the definite integral  f (x ) dx .
 2
Q.21 The graph of the derivative f ' of a continuous function f
is shown with f (0) = 0. If
(i) f is monotomic increasing in the interval
[a, b)(c, d)(e, f] and decreasing in (p, q)(r, s).
(ii) f has a local minima at x = x1 and x = x2.
(iii) f is concave up in (l, m)  (n, t]
(iv) f has inflection point at x = k
(v) number of critical points of y = f (x) is 'w'.
Find the value of (a + b + c + d + e) + (p + q + r + s) + (l + m + n) + (x1 + x2) + (k + w).
3 2 5
Q.22 Find the set of value of m for the cubic x3 – x + = log1 4 (m) has 3 distinct solutions.
2 2
2
Q.23 Find the positive value of k for the value of the definite integral  cos x  kx dx is minimised.
0
Q.24 The value of 'a' for which f (x) = x3 + 3 (a  7)x2 + 3 (a2  9)x  1 have a positive point of maximum lies
in the interval (a1, a2)  (a3, a4). Find the value of a2 + 11a3 + 70a4.
Q.25 Find the magnitude of the vertex angle ‘’ of an isosceles triangle of the given area ‘A’ such that the
radius ‘r’ of the circle inscribed into the triangle is the maximum.
Q.26 The function f (x) defined for all real numbers x has the following properties
(i) f (0) = 0, f (2) = 2 and f ' (x) = k(2x – x2)e –x for some constant k > 0. Find
(a) the intervals on which f is increasing and decreasing and any local maximum or minimum values.
(b) the intervals on which the graph f is concave down and concave up.
(c) the function f (x) and plot its graph.
Q.27 Let f (x) be a non-constant twice differentiable function defined on (– , ) such that f(x) = f(4 –
x) and f(x) = 0 has atleast 2 distinct repeated roots in (2, 4), then find the minimum number of roots
of f "(x) = 0 in [0, 4].
Q.28 Let f (x) = e(p +1)x – ex for real number p > 0.
(a) Find the value of x = sp for which f (x) is minimum.
t 1
tx
(b) Let g (t) =  f ( x )e dx . Find the value of t = tp, for which g (t) is minimum.
t
p
(c) Use the fact that 1  p  e  1  1  p  p 2 (0 < p  1) , find the value of Lim (s p  t p )
2 p 2 p 0

Campus-I : F-106, Road No.2, Indraprastha Industrial Area, Kota Ph : 2420151 / 52, 9166876626
Campus-II : A-10, Jawahar Nagar Main Road, Kota Ph : 2406123 / 24
44
Mathematics (JEE-Advanced) Application of Derivative
EXERCISE - III(A)
(TANGENT & NORMAL)
Q.1 A function y = f(x) has a second order derivative f (x) = 6( x – 1). If its graph passes through the point
(2,1) and at that point the tangent to the graph is y = 3x – 5, then the function, is [AIEEE 2004]
(A) (x– 1)2 (B) (x– 1)3 (C) (x+1)3 (D) (x+1)2

Q.2 The normal to the curve x = a (1+ cos ), y = a sin  at ‘’ always passes through the fixed point
(A) (a, 0) (B) (0, a) (C) (0,0) (D) (a, a)[AIEEE 2004]

Q.3 The normal to the curve x = a (cos  +  sin ), y = a (sin  –  cos ) at any point '' is such that
[AIEEE-2005]

(A) it passes through the origin (B) it makes angle +  with the x-axis
2
  
(C) it passes through  a 2 ,a  (D) it is at a constant distance from the origin.
 
Q.4 A spherical iron ball 10 cm in radius is coated with a layer of ice of uniform thickness that melts at a rate
of 50 cm3/min. When the thickness of ice is 5 cm, then the rate of which the thickness of ice decreases,
is - [AIEEE-2005]
1 1 1 5
(A) 36 cm/min. (B) 18 cm/min. (C) 54 cm/min. (D) 6  cm/min.
Q.5 Angle between the tangents to the curve y = x2 – 5x + 6 at the points (2, 0) and (3, 0) is
   
(A) (B) 6 (C) (D) 3 [AIEEE 2006]
2 4
Q.6 The shortest distance between the line y – x = 1 and the curve x = y2 is [AIEEE 2009]
3 2 2 3 3 2 3
(A) (B) (C) (D)
8 8 5 4
4
Q.7 The equation of the tangent to the curve y  x  , that is parallel to the x-axis, is [AIEEE 2010]
x2
(A) y = 1 (B) y = 2 (C) y = 3 (D) y = 0
Q.8 A spherical balloon is filled with 4500p cubic meters of helium gas. If a leak in the ballon causes the gas
to escape at the rate of 72cubic meters per minute, then the rate (in meters per minute) at which the
radius of the balloon decreases 49 minutes after the leakage began is : [AIEEE 2012]
(A) 9/7 (B) 7/9 (C) 2/9 (D) 9/2
Q.9 At present, a firm is manufacturing 2000 items. It is estimated that the rate of change of production P
dp
w.r.t. additional number of worker x is given by =100 – 12 x . If the firm employs 25 more
dx
workers, then the new level of production of items is : [JEE-Main 2013]
(A) 4500 (B) 2500 (C) 3000 (D) 3500
x
Q.10 The intercepts on x-axis made by tangents to the curve, y   | t | dt, x  R , which are parallel to the
0
line y = 2x, are equal to : [JEE-Main 2013]
(A) ± 4 (B) ± 1 (C) ± 2 (D) ± 3

Campus-I : F-106, Road No.2, Indraprastha Industrial Area, Kota Ph : 2420151 / 52, 9166876626
Campus-II : A-10, Jawahar Nagar Main Road, Kota Ph : 2406123 / 24
45
Mathematics (JEE-Advanced) Application of Derivative
Q.11 If f and g are differentiable functions in [0, 1] satisfying f(0) = 2 = g(1), g(0) = 0 and f(1) = 6, then for
some c  ]0, 1[ [JEE Main 2014]
(A) 2f '(c) = g'(c) (B) 2f ' (c) = 3g'(c) (C) f '(c) = g'(c) (D) f '(c) = 2g'(c)
2 2
Q.12 The normal to the curve, x + 2xy–3y = 0, at (1, 1) : [JEE Main 2015]
(A) does not meet the curve again. (B) meets the curve again in the second quadrant.
(C) meets the curve again in the third quadrant. (D) meets the curve again in the fourth quadrant.

(MONOTONOCITY)
Q.1 If 2a + 3b + 6c = 0, then at least one root of the equation ax2 + bx + c = 0 lies in the interval-
(A) (0,1) (B) (1,2) (C) (2,3) (D) (1,3) [AIEEE 2004]
Q.2 If the equation anxn + an–1 xn–1 + .... + a1x = 0 ; a1  0, n  2, has a positive root x = , then the
equation nanxn–1 + (n – 1) an–1 xn–2 + .... + a1 = 0 has a positive root, which is [AIEEE-2005]
(A) greater than  (B) smaller than 
(C) greater than or equal to  (D) equal to 
Q.3 Let f be differentiable for all x. If f(1) = – 2 and f (x)  2 for x  [1, 6], then [AIEEE-2005]
(A) f(6)  8 (B) f(6) < 8 (C) f(6) < 5 (D) f(6) = 5
Q.4 A value of C for which the conclusion of Mean Value Theorem holds for the function f(x) = logex on the
interval [1, 3] is- [AIEEE 2007]
1
(A) 2log3 e (B) loge3 (C) log3e (D) loge3
2
Q.5 A function is matched below against an interval where it is supposed to be increasing. Which of the
following pairs is incorrectly matched ? [AIEEE-2005]
interval function
(A) (–, ) x – 3x2 + 3x + 3
3

(B) [2, ) 2x3 – 3x2 – 12x + 6


 1
(C)  ,  3x2 – 2x + 1
 3
(D) (–, –4] x3 + 6x2 + 6

Q.6 The function f(x) = tan-1 (sinx + cosx) is an increasing function in- [AIEEE 2007]
(A) (/4,  /2) (B) (–/2,  /4) (C) (0,  /2) (D) (–/2,  /2)
1
Q.7 Let f : R  R be a continuous functions defined by f (x) = [AIEEE 2010]
e  2e – x
x

Statement-1 : f (c) = 1/3, for some e  R.


1
Statement-2 : 0 < f(x)  , for all x  R
2 2
(A) Statement-1 is true, statement-2 is true and statement-2 is correct explanation for statement-1.
(B) Statement-1 is true, statement-2 is true and statement-2 is NOT the correct explanation for statement-1.
(C) Statement-1 is true, statement-2 is false.
(D) Statement-1 is false, statement-2 is true
Q.8 The real number k for which the equation 2x3 + 3x + k = 0 has two distinct real roots in [0, 1]
(A) lies between 2 and 3 (B) lies between –1 and 0 [AIEEE 2013]
(C) does not exist (D) lies between 1 and 2

Campus-I : F-106, Road No.2, Indraprastha Industrial Area, Kota Ph : 2420151 / 52, 9166876626
Campus-II : A-10, Jawahar Nagar Main Road, Kota Ph : 2406123 / 24
46
Mathematics (JEE-Advanced) Application of Derivative
(MAXIMA & MINIMA)
Q.1 If the function f(x) = 2x3 – 9ax2 + 12a2 x + 1, where a > 0, attains its maximum and minimum at p and
q respectively such that p2 = q, then a equals [AIEEE 2003]
(A) 1/2 (B) 3 (C) 1 (D) 2

Q.2 The real number x when added to its inverse gives the minimum value of the sum at x equal to-
[AIEEE 2003]
(A) – 2 (B) 2 (C) 1 (D) – 1
x 2
Q.3 The function f(x) = + has a local minimum at – [AIEEE 2006]
2 x
(A) x = – 2 (B) x = 0 (C) x = 1 (D) x = 2
Q.4 A triangular park is enclosed on two sides by a fence and on the third side by a straight river bank. The
two sides having fence are of same length x. The maximum area enclosed by the park is
[AIEEE 2006]
x3 1 3
(A) (B) x2 (C)  x2 (D) x2
8 2 2
Q.5 If p and q are positive real numbers such that p2 + q2 = 1, then the maximum value of (p + q) is-
[AIEEE 2007]
1
(A) 2 (B) 1/2 (C) (D) 2
2
Q.6 Suppose the cubic x3 – px + q has three distinct real roots where p > 0 and q > 0. Then which one of
the following holds ? [AIEEE 2008]
p p
(A) The cubic has minima at – and maxima at
3 3
p p
(B) The cubic has minima at both and –
3 3
p p
(C) The cubic has maxima at both and –
3 3
p p
(D) The cubic has minima at and maxima at –
3 3
Q.7 Given P(x) = x4 +ax3 + bx2 +cx + d such that x = 0 is the only real root of P’ (x) =0. If P(–1) < P(1),
then in the interval [–1,1] - [AIEEE 2009]
(A) P (–1) is the minimum and P(1) is the maximum of P
(B) P(–1) is not minimum but P(1) is the maximum of P
(C) P(–1) is the minimum but P(1) is not the maximum of P
(D) Neither P(–1) is the minimum nor P(1) is the maximum of P
Q.8 Let f ; R  R be defined by [AIEEE 2010]
k – 2x , if x  –1
f (x) = 
 2 x  3, if x  –1
If f has a local minimum at x = – 1, then a possible value of k is
(A) 0 (B) –1/2 (C) –1 (D) 1

Campus-I : F-106, Road No.2, Indraprastha Industrial Area, Kota Ph : 2420151 / 52, 9166876626
Campus-II : A-10, Jawahar Nagar Main Road, Kota Ph : 2406123 / 24
47
Mathematics (JEE-Advanced) Application of Derivative
x
 5  t sin t dt . Then f has
Q.9 For x   0,  , define f(x) =
 2 
 [AIEEE 2011]
0

(A) Local minimum at  and 2 (B) Local minimum at  and local maximum at 2
(C) Local minimum at  and local minimum at 2 (D) Local minimum at  and 2
Q.10 Let a, b  R be such that the function f given by f (x) = n|x| + bx2 + ax, x  0 has extreme values at
x = – 1 and x = 2 [AIEEE 2012]
Statement-1 : f has local maximum at x = –1 and at x = 2.
1 –1
Statement-2 : a = and b 
2 4
(A) Statement-1 is true, statement-2 is true and statement-2 is correct explanation for statement-1.
(B) Statement-1 is true, statement-2 is true and statement-2 is NOT the correct explanation for statement-1.
(C) Statement-1 is true, statement-2 is false.
(D) Statement-1 is false, statement-2 is true
Q.11 If x = – 1 and x = 2 are extreme points of f(x) =  log |x| + bx2 + x, then [JEE Main 2014]
(A)  = – 6,  = 1/2 (B)  = – 6,  = –1/2
(C)  = 2,  = –1/2 (D)  = 2,  = 1/2
Q.12 Let f(x) be a polynomial of degree four having extreme values at x = 1 and x = 2.
 f ( x) 
If lim 1  2  = 3, then f(2) is equal to : [JEE Main 2015]
x 0
 x 
(A) –8 (B) –4 (C) 0 (D) 4

EXERCISE - III(B)
(TANGENT & NORMAL)
Q.1 The point(s) on the curve y3 + 3x2 = 12y where the tangent is vertical, is(are) [JEE 2002 (Scr.)]

 4   11   4 
(A)   ,  2 (B)   3 , 1 (C) (0, 0) (D)   , 2
 3     3 

Q.2 Tangent to the curve y = x2 + 6 at a point P (1, 7) touches the circle x2 + y2 + 16x + 12y + c = 0 at a
point Q. Then the coordinates of Q are [JEE 2005 (Scr.)]
(A) (– 6, –11) (B) (–9, –13) (C) (– 10, – 15) (D) (–6, –7)
Q.3 The tangent to the curve y = ex drawn at the point (c, ec) intersects the line joining the points
(c – 1, ec – 1) and (c + 1, ec + 1)
(A) on the left of x = c (B) on the right of x = c
(C) at no point (D) at all points [JEE 2007]

Campus-I : F-106, Road No.2, Indraprastha Industrial Area, Kota Ph : 2420151 / 52, 9166876626
Campus-II : A-10, Jawahar Nagar Main Road, Kota Ph : 2406123 / 24
48
Mathematics (JEE-Advanced) Application of Derivative
(MONOTONOCITY)
 
Q.1 (a) Using the relation 2(1 – cosx) < x2 , x  0 or otherwise, prove that sin (tanx) > x ,  x 0,  .
 4
(b) Let f : [0, 4]  R be a differentiable function.
(i) Show that there exist a, b  [0, 4], (f (4))2 – (f (0))2 = 8 f (a) f (b)
(ii) Show that there exist ,  with ,   (0, 2) such that
4
 f(t) dt = 2 ( f (2) +  f (2) ) [JEE 2003 (Mains)]
0

x  lnx , x  0

Q.2(a) Let f (x) =  . Rolle’s theorem is applicable to f for x [0, 1], if =
0, x0
(A) –2 (B) –1 (C) 0 (D) 1/2

f (x 2 )  f ( x )
(b) If f is a strictly increasing function, then Lim is equal to
x  0 f ( x )  f ( 0)

(A) 0 (B) 1 (C) –1 (D) 2


[JEE 2004 (Scr)]
Q.3 If p (x) = 51x101 – 2323x100 – 45x + 1035, using Rolle's theorem, prove that at least one root of p(x)
lies between (451/100, 46). [JEE 2004]

Q.4 If f (x) is a twice differentiable function and given that f(1) = 1, f(2) = 4, f(3) = 9, then
(A) f '' (x) = 2, for  x  (1, 3) (B) f '' (x) = f ' (x) = 2, for some x  (2, 3)
(C) f '' (x) = 3, for  x  (2, 3) (D) f '' (x) = 2, for some x  (1, 3)
[JEE 2005 (Scr)]
Q.5(a) Let f (x) = 2 + cos x for all real x.
Statement-1: For each real t, there exists a point 'c' in [t, t + ] such that f ' (c) = 0.
because
Statement-2: f (t) = f (t + 2) for each real t.
(A) Statement-1 is true, statement-2 is true; statement-2 is correct explanation for statement-1.
(B) Statement-1 is true, statement-2 is true; statement-2 is NOT a correct explanation for statement-1.
(C) Statement-1 is true, statement-2 is false.
(D) Statement-1 is false, statement-2 is true. [JEE 2007]
Paragraph [JEE 2007]
Q.5(b) If a continuous function f defined on the real line R, assumes positive and negative values in R then the
equation f (x) = 0 has a root in R. For example, if it is known that a continuous function f on R is
positive at some point and its minimum value is negative then the equation f (x) = 0 has a root in R.
Consider f (x) = kex – x for all real x where k is a real constant.
Campus-I : F-106, Road No.2, Indraprastha Industrial Area, Kota Ph : 2420151 / 52, 9166876626
Campus-II : A-10, Jawahar Nagar Main Road, Kota Ph : 2406123 / 24
49
Mathematics (JEE-Advanced) Application of Derivative
(i) The line y = x meets y = kex for k  0 at
(A) no point (B) one point (C) two points (D) more than two points
(ii) The positive value of k for which kex – x = 0 has only one root is
(A) 1/e (B) 1 (C) e (D) loge2
(iii) For k > 0, the set of all values of k for which kex – x = 0 has two distinct roots is

(A) 0, 1 e (B) 1 e , 1 (C) 1 e ,   (D) (0, 1)

Match the column. [JEE 2007]


Q.5(c) In the following [x] denotes the greatest integer less than or equal to x.
Match the functions in Column I with the properties in Column II.
Column I Column II
(A) x|x| (P) continuous in (–1, 1)

(B) |x| (Q) differentiable in (–1, 1)


(C) x + [x] (R) strictly increasing in (–1, 1)
(D) |x–1|+|x+1| (S) non differentiable at least at one point in (–1, 1)

   
Q.6(a) Let the function g : (– , )    ,  be given by g (u) = 2 tan–1(eu) – . Then, g is
 2 2 2

(A) even and is strictly increasing in (0, )


(B) odd and is strictly decreasing in (– , )
(C) odd and is strictly increasing in (– , )
(D) neither even nor odd, but is strictly increasing in (– , )
Q.6(b) Let f (x) be a non-constant twice differentiable function defined on (–, ) such that f (x) = f (1 – x)
and f ' 1 4  = 0. Then
(A) f ''(x) vanishes at least twice on [0, 1] (B) f ' 1 2  = 0
1/ 2 1/ 2 1
 1
(C)  f  x   sin x dx = 0 (D)  f ( t ) esin t dt =  f (1  t ) e sin t dt
1 / 2  2 0 1/ 2
[JEE 2008]

1
Q.7 For the function f ( x )  x cos , x  1,
x
(A) for at least one x in the interval [1, ), f(x + 2) – f(x) < 2

(B) xlim

f (x) = 1
(C) for all x in the interval [1, ), f(x + 2) – f(x) > 2
(D) f (x) is strictly decreasing in the interval [1, ) [JEE 2009]

Campus-I : F-106, Road No.2, Indraprastha Industrial Area, Kota Ph : 2420151 / 52, 9166876626
Campus-II : A-10, Jawahar Nagar Main Road, Kota Ph : 2406123 / 24
50
Mathematics (JEE-Advanced) Application of Derivative
x

Q.8(a) Let f be a real valued function defined on the interval (0, ) by f (x) = ln x +  1  sin t dt . Then which
0
of the following statement(s) is/are true?
(A) f '' (x) exists for all x  (0, )
(B) f ' (x) exists for all x  (0, ) and f ' is continuous on (0, ) but not differentiable on (0, ).
(C) there exists  > 1 such that | f ' (x) | < | f (x) | for all x  (, )
(D) there exists  > 0 such that | f (x) | + | f ' (x) |   for all x  (0, )
(b) Consider the polynomial
f(x) = l + 2x + 3x2 + 4x3. Let s be the sum of all distinct real roots of f(x) and let t = | s |.

(i) The real number s lies in the interval

 1   3  3 1  1
(A)   , 0  (B)   11,   (C)   ,   (D)  0, 
 4   4  4 2  4

(ii) The area bounded by the curve y = f(x) and the lines x = 0, y = 0 and x = t, lies in the interval

3   21 11   21 
A)  , 3  (B)  ,  (C) (9, 10) (D)  0, 
4   64 16   64 

(iii) The function f (x) is

 1  1 
(A) increasing in   t ,   and decreasing in   , t 
 4  4 

 1  1 
(B) decreasing in   t,   and increasing in   , t 
 4  4 

(C) increasing in (– t, t)
(D) decreasing in (– t, t) [JEE 2010]

x
 2 ( t  1) 
Q.9 Let f(x) = (1 – x)2 sin2x + x2  x  R and let g(x) =    ln t  f ( t ) dt  x  (1, ).
1
t  1 

(a) Consider the statements :


P : There exists some x  R such that f(x) + 2x = 2 (1 + x2).
Q : There exist some x  R such that 2f(x) + 1 = 2x (1 + x).
Then
(A) both P and Q are true (B) P is true and Q is false
(C) P is false and Q is true (D) both P and Q are false

Campus-I : F-106, Road No.2, Indraprastha Industrial Area, Kota Ph : 2420151 / 52, 9166876626
Campus-II : A-10, Jawahar Nagar Main Road, Kota Ph : 2406123 / 24
51
Mathematics (JEE-Advanced) Application of Derivative
(b) Which of the following is true?
(A) g is increasing on (1, ).
(B) g is decreasing on (1, ).
(C) g is increasing on (1, 2) and decreasing on (2, ).
(D) g is decreasing on (1, 2) and increasing on (2, ). [JEE 2012, 3+3]

1 
Q.10 Let f :  , 1  R (the set of all real numbers) be a positive, non-constant and differentiable function
2 
1
1
such that f'(x) < 2 f(x) and f   = 1. Then the value of  f ( x )dx lies in the interval [JEE Adv. 2013]
 2 1/ 2

 e –1   e –1
(A) (2e – 1, 2e) (B) (e – 1, 2e –1) (C)  , e – 1 (D)  0, 
 2   2 

Q.11 Let f(x) = x sinx, x > 0. Then for all natural numbers n, f '(x) vanishes at [JEE Adv. 2013]
 1
(A) a unique point in the interval  n , n  
 2

 1 
(B) a unique point in the interval  n  , n  1
 2 
(C) a unique point in the interval (n, n +1)
(D) two points in the interval (n, n + 1)
Q.12 For every pair of continuous functions f, g:[0, 1] R such that [JEE ADV 2014]
max {f(x) : x  [0,1]} = max {g(x) : x  [0, 1]}, the correct statement(s) is (are) :
(A) (f(c))2 + 3f(c) = (g(c))2 + 3g(c) for some c  [0, 1]
(B) (f(c))2 + f(c) = (g(c))2 + 3g(c) for some c  [0, 1]
(C) (f(c))2 + 3f(c) = (g(c))2 + g(c) for some c  [0, 1]
(D) (f(c))2 = (g(c))2 for some c  [0, 1]
Q.13 Let a  R and let f : R R be given by f(x) = x5 – 5x + a. Then [JEE ADV 2014]
(A) f(x) has three real roots if a > 4 (B) f(x) has only one real root if a > 4
(C) f(x) has three real roots if a < – 4 (D) f(x) has three real roots if – 4 < a < 4

(MAXIMA & MINIMA)


Q.1(a) Find a point on the curve x2 + 2y2 = 6 whose distance from the line x + y = 7, is minimum.
(b) For a circle x2 + y2 = r2, find the value of ‘r’ for which the area enclosed by the tangents drawn from
the point P(6, 8) to the circle and the chord of contact is maximum. [JEE 2003, Mains]
Q.2(a) Let f (x) = x3 + bx2 + cx + d, 0 < b2 < c. Then f
(A) is bounded (B) has a local maxima
(C) has a local minima (D) is strictly increasing [JEE 2004 (Scr.)]

3x · (x  1)  
(b) Prove that sin x  2 x   x  0,  . (Justify the inequality, if any used).
  2
[JEE 2004]

Campus-I : F-106, Road No.2, Indraprastha Industrial Area, Kota Ph : 2420151 / 52, 9166876626
Campus-II : A-10, Jawahar Nagar Main Road, Kota Ph : 2406123 / 24
52
Mathematics (JEE-Advanced) Application of Derivative
Q.3 If P(x) be a polynomial of degree 3 satisfying P(–1) = 10, P(1) = – 6 and P(x) has maximum at
x = – 1 and P'(x) has minima at x = 1. Find the distance between the local maximum and local minimum
of the curve. [JEE 2005 (Mains)]
Q.4(a) Let f (x) be a polynomial of degree 3 having a local maximum at x = – 1. If f (–1) = 2, f (3) = 18,
and f ' (x) has a local minimum at x = 0, then
(A) distance between (–1, 2) and (a, f (A)), which are the points of local maximum and local minimum
on the curve y = f (x) is 2 5

(B) f (x) is a decreasing function for 1  x  2 5

(C) f '(x) has a local maximum at x = 2 5


(D) f (x) has a local minimum at x = 1

x , 0  x 1 x
 x 1
(b) Let f (x) = 2  e , 1  x  2 and g (x) =  f t  dt , 0  x  3. Then,
x  e, 2  x  3 0

(A) g has a local minimum at x = e and a local maximum at x = 1 + ln 2


(B) g has a local minimum at x = 2 and a local maximum at x = 1
(C) g does not have a local maximum
(D) g does not have a local minimum

(c) Let f : R  R be a twice differentiable function such that f (A) = 0, f (b) = 2, f (c) = – 1, f (d) = 2,
f (e) = 0, where a < b < c < d < e. Determine the minimum number of zeroes of

g( x )  f ' ( x ) 2  f (x ).f " ( x ) in the interval [a, e]. [JEE 2006]

(2  x )3 ,  3  x  1
Q.5(a) The total number of local maxima and local minima of the function f (x) =  2 / 3 is
x , 1  x  2

(A) 0 (B) 1 (C) 2 (D) 3


(b) Comprehension:
Consider the function f : (– , )  (– , ) defined by

x 2  ax  1
f (x)  2 ,0<a<2
x  ax  1
(i) Which of the following is true?
(A) (2 + a)2 f '' (1) + (2 – a)2 f '' (– 1) = 0 (B) (2 – a)2 f '' (1) – (2 + a)2 f '' (– 1) = 0
(C) f ' (1) f ' (–1) = (2 – a)2 (D) f ' (1) f ' (–1) = – (2 + a)2
(ii) Which of the following is true?
(A) f (x) is decreasing on (–1, 1) and has a local minimum at x = 1
(B) f (x) is increasing on (–1, 1) and has a local maximum at x = 1
(C) f (x) is increasing on (–1, 1) but has neither a local maximum and nor a local minimum at x=1.
(D) f (x) is decreasing on (–1, 1) but has neither a local maximum and nor a local minimum at x = 1.
Campus-I : F-106, Road No.2, Indraprastha Industrial Area, Kota Ph : 2420151 / 52, 9166876626
Campus-II : A-10, Jawahar Nagar Main Road, Kota Ph : 2406123 / 24
53
Mathematics (JEE-Advanced) Application of Derivative
ex
f ' (t)
(iii) Let g (x) =  1  t 2 dt
0

Which of the following is true?


(A) g ' (x) is positive on (– , 0) and negative on (0, )
(B) g ' (x) is negative on (– , 0) and positive on (0, )
(C) g ' (x) changes sign on both (– , 0) and (0, )
(D) g ' (x) does not change sign on (– , ) [JEE 2008]

 p( x ) 
Q.6(a) Let p(x) be a polynomial of degree 4 having extremum at x = 1, 2 and Lim 1  2   2 . Then the
x 0  x 
value of p(2) is

(b) 
The maximum value of the function f(x) = 2x3 – 15x2 + 36x – 48 on the set A  x | x 2  20  9x is 
[JEE 2009]
2 2
x x
Q.7(a) Let f, g and h be real-valued functions defined on the interval [0, 1] by f (x) = e  e ,
2 2 2 2
x x
g (x) =  x e  e and h(x) = x 2 e x  e  x . If a, b and c denote respectively, the absolute maximum
of f, g and h on [0, 1], then
(A) a = b and c  b (B) a = c and a  b (C) a  b and c  b (D) a = b = c
(b) Let f be a function defined on R (the set of all real numbers) such that
f (x) = 2010(x - 2009)(x - 2010)2(x - 2011)3 (x - 2012)4, for all x  R.
If g is a function defined on R with values in the interval (0, ) such that
f(x) = ln (g (x)), for all x  R,
then the number of points in R at which g has a local maximum is [JEE 2010]
Q.8 The number of distinct real roots of x4 – 4x3 +12x2 + x – 1 = 0 is [JEE 2011]
x
2
Q.9(a) If f (x) =  e t ( t  2)( t  3) dt for all x  (0, ), then
0

(A) f has a local maximum at x = 2


(B) f is decreasing on (2, 3)
(C) there exists some c  (0, ) such that f '' (c) = 0
(D) f has a local minimum at x = 3.
(b) Let f : R  R be defined as f (x) = | x | + | x2 – 1 |. The total number of points at which f attains either
a local maximum or a local minimum is
(c) Let p (x) be a real polynomial of least degree which has a local maximum at x = 1 and a local minimum
at x = 3. If p(1) = 6 and p(3) = 2, then p'(0) is [JEE 2012]
Q.10 The number pf points in (–, ), for which x2 – x sin x – cosx = 0, is [JEE. Adv. 2013]
(A) 6 (B) 4 (C) 2 (D) 0

Campus-I : F-106, Road No.2, Indraprastha Industrial Area, Kota Ph : 2420151 / 52, 9166876626
Campus-II : A-10, Jawahar Nagar Main Road, Kota Ph : 2406123 / 24
54
Mathematics (JEE-Advanced) Application of Derivative
Q.11 A rectangular sheet of fixed perimeter with sides having their lengths in the ratio 8 : 15 is converted into
an open rectangular box by folding after removing squares of equal area from all four corners. If the
total area of removed squares in 100, the resulting box has maximum value of the sides of the
rectangular sheet are [JEE. Adv. 2013]
(A) 24 (B) 32 (C) 45 (D) 60
Q.12 The function f(x) = 2|x| + |x + 2| – ||x + 2| – 2|x|| has a local minimum or a local maximum at x =

–2 2
(A) –2 (B) (C) 2 (D) [JEE. Adv. 2013]
3 3

Paragraph for questions 13 and 14 [JEE. Adv. 2013]


Let f : [0, 1]  R (the set of all real numbers) be a function. Suppose the function f is twice differentiable,
f(0) = f(1) = 0 and satisfies f ''(x) – 2f '(x) + f(x)  ex, [0, 1].
Q.13 Which of the following is true for 0 < x < 1 ?

1 1 1
(A) 0 < f(x) <  (B) – < f(x) < (C) – < f(x) < 1 (D) – < f(x) < 0
2 2 4
1
Q.14 If the function e–x f(x) assumes its minimum in the interval [0, 1] at x = , which of the following is true?
4

1 3 1
(A) f '(x) < f(x), <x< (B) f '(x) > f(x), 0 < x <
4 4 4

1 3
(C) f '(x) < f(x), 0 < x < (D) f '(x) < f(x), <x<1
4 4

Campus-I : F-106, Road No.2, Indraprastha Industrial Area, Kota Ph : 2420151 / 52, 9166876626
Campus-II : A-10, Jawahar Nagar Main Road, Kota Ph : 2406123 / 24
55
Mathematics (JEE-Advanced) Application of Derivative

ANSWER KEY
EXERCISE - I
Q.1 C Q.2 A Q.3 C Q.4 B Q.5 C
Q.6 A Q.7 B Q.8 B Q.9 A Q.10 C
Q.11 B Q.12 A Q.13 A Q.14 D Q.15 C
Q.16 D Q.17 A Q.18 C Q.19 A Q.20 A
Q.21 B Q.22 A Q.23 A Q.24 B Q.25 A
Q.26 B Q.27 C Q.28 C Q.29 A Q.30 B
Q.31 D Q.32 B Q.33 D Q.34 D Q.35 D
Q.36 C Q.37 C Q.38 A Q.39 A Q.40 C
Q.41 D Q.42 C Q.43 A Q.44 A Q.45 D
Q.46 B Q.47 D Q.48 D Q.49 C Q.50 D
Q.51 C Q.52 D Q.53 A Q.54 C Q.55 A
Q.56 C Q.57 B
EXERCISE - II(A)
(TANGENT & NORMAL)
Q.1 AC Q.2 AD Q.3 AB Q.4 CD Q.5 AB
(MONOTONOCITY)
Q.1 AD Q.2 BD Q.3 BCD Q.4 BCD Q.5 AC
Q.6 ACD Q.7 ABC Q.8 C Q.9 B Q.10 D
Q.11 (A) P,S,T ; (B) Q,R,S,T (C) Q,R,S; (D) Q,R,S

(MAXIMA & MINIMA)


Q.1 ABC Q.2 ACD Q.3 BD Q.4 BCD Q.5 ABCD
Q.6 ABCD Q.7 CD Q.8 ABD Q.9 B Q.10 D
Q.11 A Q.12 A Q.13 D Q.14 D Q.15 D
Q.16 C Q.17 C
EXERCISE - II(B)
(TANGENT & NORMAL, ROLLE'S, LMVT AND RATE MEASURE)
 1 82 · 7 3
Q.1 (0, 1) & 1,  Q.2 – Q.4 T : x – 2y = 0 ; N : 2x + y = 0
 2 3
    3 
Q.5 y1 = cos(x1 + y1) &  , 0  ,   , 0 Q.6 2a Q.7 5
2   2 
1 1 3
Q.9 f (x) = – ; x – 4y = 2 Q.10 1/16 Q.11 a=– , b=– , c=3
x2 2 4

ab
Q.12 x12 / 3  y12 / 3  Q.13 2e–x/2 Q.14 (i) ab = 1 (ii) two values of x are possible
c1/ 3

2 14
Q.16 6 km/h Q.17 cm/sec Q.18 Q.24 a = 3, m = 1, b = 4
4 11
Campus-I : F-106, Road No.2, Indraprastha Industrial Area, Kota Ph : 2420151 / 52, 9166876626
Campus-II : A-10, Jawahar Nagar Main Road, Kota Ph : 2406123 / 24
56
Mathematics (JEE-Advanced) Application of Derivative
[Monotonocity]
Q.1 (a) I in (2 , ) & D in ( , 2) (b) I in (1 , ) & D in (  , 0)  (0 , 1)
(c) I in (0, 2) & D in ( , )  (2 , )
Q.2 x  (–2, 0)  (2, ), f(x) is decreasing
Q.3 (a) I in [0, 3/4)  (7/4 , 2 ] & D in (3/4 , 7 /4)
(b) I in [0 , /6)  (/2 , 5/6)  (3/2 , 2 ] & D in (/6 , /2)  (5/6, 3 /2)]
(c) I in [0, /2)  (3/2, 2] and D in (/2, 3/2)
Q.4 (0, 1/3)  (1, 5) Q.5 continuous but not diff. at x = 1
Q.6   
(a) a   ,  2  5  5 ,  ] 
(b) If a > 6 because then even if cos 6x & cos 5x are both 1 then also f  (x) shell be +ve.]
 7 5
(c) a    ,  
 2 2
Q.7 (a) (/6)+(1/2)ln 3, (/3) – (1/2)ln 3
(b) Maximum at x = 1 and f (–1) = 18; Minimum at x = 1/8 and f (1/8) = – 9/4
(c) 2 & 10
(b  a)3
Q.8 x  n & a1 Q.9 – 10 Q.10 Ymin =
4
Q.11 [ 7,  1)  [2, 3] Q.12  in (3, ) and  in (1, 3)
Q.13 a  0 Q.16 0 < c < 1/9
[Maxima & Minima]
Q.1 f(x) = x3 + x2  x + 2 Q.2 max. at x = 1 ; f(1) = 0 , min. at x = 7/5 ; f(7/5) =  108/3125
Q.3 (a) Max at x = 2 , Max value = 2 , Min. at x = 0 , Min value = 0
(b) Max at x = /6 & also at x = 5 /6 and Max value = 3/2 , Min at x = /2 , Min value =  3
1 5 1
Q.4 a= ; b =  ; f(x) = (x2  5x + 8) Q.5 6
4 4 4
Q.7 3x + 4y – 9 = 0 ; 3x – 4y + 9 = 0 Q.8 25
 
Q.9 16 Q.10 58 Q.11Pmax = a  1  cosec 
 2
1
Q.12 75 3 sq. units Q.13 h (x)]max = h(1) = e + = c , a = b = c
e
6
Q.14 x = 0 or x = – 4 Q.16 Q.18 32
108
2
Q.19 (a) f is continuous at x = 0 ; (b) – ; (c) does not exist, does not exist ; (d) pt. of inflection x = 1 ]
e
 3
Q.20 (a) x = – 2, – , 0, , 2, (b) no inflection point, (c) maxima at x = and – and no minima, (d)
2 2
3 
x= and x = – , (e) –  ln 2
2 2
 1 1 2 2   
Q.21 74 Q.22 m   ,  Q.23 k = cos 
 32 16   2 2 
Q.24 320 Q.25  = 60°

Campus-I : F-106, Road No.2, Indraprastha Industrial Area, Kota Ph : 2420151 / 52, 9166876626
Campus-II : A-10, Jawahar Nagar Main Road, Kota Ph : 2406123 / 24
57
Mathematics (JEE-Advanced) Application of Derivative
Q.26 (a) increasing in (0, 2) and decreasing in (–, 0)  (2, ), Local minimum value = 0 and local maximum
value = 2
(b) concave  for (– , 2 – 2 )  (2 + 2 , ) and concave  in (2 – 2 ), (2 + 2)
1 2· x 2
(c) f (x) = e ·x ]
2
p
 ln (p  1) 1  ( p  1)(e  1)  1
Q.27 6 Q.28 (i) x = sp = (ii) tp = – ln  p  (iii)  =
p p   2
EXERCISE - III(A)
(TANGENT & NORMAL)
Q.1 B Q.2 A Q.3 BD Q.4 B Q.5 A
Q.6 A Q.7 C Q.8 C Q.9 D Q.10 B
Q.11 D Q.12 D
(MONOTONOCITY)
Q.1 A Q.2 B Q.3 A Q.4 A Q.5 C
Q.6 B Q.7 A Q.8 C

(MAXIMA & MINIMA)


Q.1 D Q.2 C Q.3 D Q.4 B Q.5 D
Q.6 D Q.7 B Q.8 C Q.9 C Q.10 B
Q.11 C Q.12 C
EXERCISE - III(B)
(TANGENT & NORMAL)
Q.1 D Q.2 D Q.3 A
(MONOTONOCITY)
Q.2 (a) D; (b) C Q.4 D
Q.5 (a) B; (b) (i) B, (ii) A, (iii) A; (c) (A) P,Q,R; (B) P,S; (C) R,S; (D) P, Q
Q.6 (a) C, (b) A, B, C, D Q.7 B, C, D Q.8 (a) B, C ; (b) (i) C ; (ii) A ; (iii) B
Q.9 (a) C ; (b) B Q.10 D Q.11 BC
Q.12 AD Q.13 BD
(MAXIMA & MINIMA)
Q.1 (a) (2, 1) ; (b) 5 Q.2 (a) D Q.3 4 65
Q.4 (a) A, D; (b) A; (c) 6 Q.5 (a) C; (b) (i) A, (ii) A, (iii) B Q.6 (a) 0; (b) 7
Q.7 (a) D, (b) 1 Q.8 2 Q.9 (a) ABCD; (b) 5 ; (C) 9
Q.10 C Q.11 AC Q.12 A, B Q.13 D Q.14 C

Campus-I : F-106, Road No.2, Indraprastha Industrial Area, Kota Ph : 2420151 / 52, 9166876626
Campus-II : A-10, Jawahar Nagar Main Road, Kota Ph : 2406123 / 24
58

You might also like